Techniques d’algèbre

image
Récapitulatifs de techniques classiques en algèbre.

sans techniqu’,un don n’est rien qu’un’ sal’ manie. Georges Brassens (1921-1981)

Trigonométrie

Dans ce paragraphe, nous allons voir les formules de trigonométrie à connaître par cœ ur ou à savoir retrouver rapidement. Il est aussi important de comprendre à quoi servent ces formules.

Lecture du cercle trigonométrique

Il faut savoir interpréter pour un angle \(\theta\), son sinus, son cosinus, sa tangente et sa cotangente géométriquement sur le cercle trigonométrique :

Multimédia: animation avec l’angle qui varie

On représente une demi-droite issue de l’origine faisant un angle \(\theta\) avec l’axe \((Ox)\), cette demi-droite coupe le cercle unité en un point \(A\), coupe la tangente au cercle au point \(B=(1,0)\) en un point \(P\) et la tangente au cercle au point \(C=(0,1)\) en un point \(Q\). On interprète \(\sin\theta\), \(\cos\theta\), \(\tan\theta\), \(\mathop{\mathrm{cotan}}\theta = \dfrac{1}{\tan\theta}\) comme des mesures algébriques :

  • En notant \(M\) la projection orthogonale du point \(A\) sur l’axe \((Ox)\), \(\sin\theta = \overline{OM}\).

  • En notant \(N\) la projection orthogonale du point \(A\) sur l’axe \((Oy)\), \(\cos\theta = \overline{ON}\).

  • \(\tan\theta = \overline{BP}\).

  • \(\mathop{\mathrm{cotan}}\theta = \overline{CQ}\).

On a \(\cos(\pi/2-\theta)=\sin(\theta)\), \(\sin(\pi/2-\theta)=\cos(\theta)\), \(\tan(\pi/2-\theta) = \mathop{\mathrm{cotan}}(\theta)\) et \(\mathop{\mathrm{cotan}}(\pi/2-\theta) = \tan(\theta)\). Pour retrouver d’autres simplifications, remarquons que si l’on choisit sur le dessin un angle \(\theta\) petit et positif, \(\sin\theta\) est petit positif, \(\cos\theta\) est proche de \(1\) positif, \(\tan\theta\) est petit positif et \(\mathop{\mathrm{cotan}}\theta\) est grand et positif.

On retrouve sans calculs les simplifications de \(\sin(k\pi \pm \theta)\), \(\sin(k\pi+\pi/2 \pm \theta)\), \(\cos(k\pi \pm \theta)\), \(\cos(k\pi+\pi/2 \pm \theta)\) en fonction de \(\sin \theta\) ou \(\cos\theta\) et les simplifications de \(\tan(k\pi \pm \theta)\), \(\tan(k\pi+\pi/2 \pm \theta)\), \(\mathop{\mathrm{cotan}}(k\pi \pm \theta)\), \(\mathop{\mathrm{cotan}}(k\pi+\pi/2 \pm \theta)\) en fonction de \(\tan\theta\) ou \(\mathop{\mathrm{cotan}}\theta\).

Simplifier \(\sin(3\pi/2 - \theta)\), \(\cos(3\pi/2-\theta)\), \(\tan(3\pi/2-\theta)\) et \(\mathop{\mathrm{cotan}}(3\pi/2-\theta)\). On prend sur le dessin un angle \(\theta\) petit positif et on représente l’angle \(3\pi/2-\theta\) :

On voit que \(\cos(3\pi/2-\theta)\) est petit et négatif et on retrouve sans calculs que \(\cos(3\pi/2-\theta) = -\sin\theta\). De même, \(\sin(3\pi/2-\theta)\) étant proche de \(-1\), il vaut \(-\cos\theta\). Puisque \(\tan(3\pi/2-\theta)\) est grand et positif, il vaut \(\mathop{\mathrm{cotan}}\theta\) et de même \(\mathop{\mathrm{cotan}}(3\pi/2-\theta) = \tan\theta\).
Simplifier en utilisant le cercle trigonométrique, \(\sin(7\pi/2+\theta)\), \(\cos(5\pi/2+\theta)\), \(\tan(\pi/2+\theta)\), \(\mathop{\mathrm{cotan}}(7\pi/2+\theta)\). \(\sin(7\pi/2+\theta)=-\cos\theta\), \(\cos(5\pi/2+\theta)=-\sin\theta\), \(\tan(\pi/2+\theta)=-\mathop{\mathrm{cotan}}\theta\), \(\mathop{\mathrm{cotan}}(7\pi/2+\theta)=-\tan\theta\).

On remarque que \(\cos(\theta+k\pi)=\pm\cos(\theta)\), \(\sin(\theta+k\pi)=\pm\sin(\theta)\) en fonction de la parité de l’entier \(k\). On vérifie la formule utile : \(\heartsuit\) \[\begin{aligned} \sin(\theta+k\pi) = (-1)^k\sin\theta \quad \cos(\theta+k\pi) = (-1)^k\cos(\theta). \end{aligned}\]

Les quatre formules fondamentales de la trigonométrie

Il n’y a que peu de formules à retenir vraiment par cœ ur en trigonométrie. Comme les méthodes que nous allons voir sont similaires en trigonométrie hyperbolique, nous les inscrivons en parallèle.

On sait exprimer \(\cos^2 x\) en fonction de \(\sin^2 x\) grâce à la formule fondamentale : \(\heartsuit\) \[\begin{aligned} \cos^2 x + \sin^2 x = 1 \quad\quad \quad\quad \quad\mathop{\mathrm{ch}}^2 x - \mathop{\mathrm{sh}}^2 x = 1. \end{aligned}\] Lorsqu’on rencontre un groupement \(\sqrt{1-y^2}\) (avec \(\lvert y \rvert \leqslant 1\)), il est souvent intéressant de poser \(y = \sin x\) ou \(y = \cos x\) pour éliminer la racine : \(\sqrt{1-\sin^2 x} = \sqrt{\cos^2 x} = \lvert \cos x \rvert\). De même pour un groupement \(\sqrt{1+y^2}\), il est intéressant de poser \(y = \mathop{\mathrm{sh}}x\) puisque \(\sqrt{1+\mathop{\mathrm{sh}}^2 x} = \sqrt{\mathop{\mathrm{ch}}^2 x} = \mathop{\mathrm{ch}}x\) et pour un groupement \(\sqrt{y^2-1}\) de poser \(y = \pm\mathop{\mathrm{ch}}x\) puisque \(\sqrt{\mathop{\mathrm{ch}}^2 x - 1} = \sqrt{\mathop{\mathrm{sh}}^2 x} = \lvert \mathop{\mathrm{sh}}x \rvert\).

Il faut connaître par cœ ur les quatre formules d’addition :

\(\heartsuit\) \[\begin{aligned} \begin{alignedat}{2} \cos(a+b) &= \cos a \cos b - \sin a \sin b &\qquad \mathop{\mathrm{ch}}(a+b) &= \mathop{\mathrm{ch}}a \mathop{\mathrm{ch}}b + \mathop{\mathrm{sh}}a \mathop{\mathrm{sh}}b \\ \cos(a-b) & =\cos a \cos b +\sin a \sin b & \qquad \mathop{\mathrm{ch}}(a-b) &= \mathop{\mathrm{ch}}a \mathop{\mathrm{ch}}b - \mathop{\mathrm{sh}}a \mathop{\mathrm{sh}}b \\ \sin(a+b) & =\sin a \cos b + \cos a \sin b &\qquad \mathop{\mathrm{sh}}(a+b) &=\mathop{\mathrm{sh}}a \mathop{\mathrm{ch}}b + \mathop{\mathrm{ch}}a \mathop{\mathrm{sh}}b \\ \sin(a-b) & =\sin a \cos b - \cos a \sin b &\qquad \mathop{\mathrm{sh}}(a-b) &=\mathop{\mathrm{sh}}a \mathop{\mathrm{ch}}b - \mathop{\mathrm{ch}}a \mathop{\mathrm{sh}}b. \end{alignedat} \end{aligned}\]

Elles permettent, en prenant \(b=a\), de trouver les deux formules importantes :

\(\heartsuit\) \[\begin{aligned} \begin{alignedat}{2} \cos 2a &= \cos^2 a - \sin^2 a = 2\cos^2 a-1 = 1-2\sin^2 a &\qquad \mathop{\mathrm{ch}}2a & = \mathop{\mathrm{ch}}^2 a + \mathop{\mathrm{sh}}^2 a = 2\mathop{\mathrm{ch}}^2 a -1 =1+2\mathop{\mathrm{sh}}^2 a \\ \sin 2a &= 2\sin a \cos a &\qquad \mathop{\mathrm{sh}}2a & = 2\mathop{\mathrm{sh}}a \mathop{\mathrm{ch}}a. \end{alignedat} \end{aligned}\]

Ces formules servent en particulier à linéariser \(\cos^2 a\) et \(\sin^2 a\) : \(\heartsuit\) \[\begin{aligned} \begin{alignedat}{2} \cos^2 a &= \dfrac{\cos 2a+1}{2} &\qquad \mathop{\mathrm{ch}}^2 a & =\dfrac{\mathop{\mathrm{ch}}2a+1}{2} \newline \sin^2 a &= \dfrac{1-\cos 2a}{2} &\qquad \mathop{\mathrm{sh}}^2 a & = \dfrac{\mathop{\mathrm{ch}} 2a-1}{2}. \end{alignedat} \end{aligned}\]

Ces formules de linéarisation sont surtout utilisées en trigonométrie classique. Pour la trigonométrie hyperbolique, les signes changent. On peut vérifier facilement une formule hyperbolique en prenant \(a=0\) et en faisant \(a\rightarrow +\infty\). Par exemple, la formule suivante est fausse : \[\mathop{\mathrm{ch}}(2a) = 1 - 2\mathop{\mathrm{sh}}^2 a\] (\(a \rightarrow +\infty\) ne va pas ! )
Ces formules servent également à exprimer \(1+\cos a\) ou \(1-\cos a\) comme un carré : \[1+\cos\theta = 2\cos^2 \dfrac{\theta}{2} \quad 1-\cos\theta = 2\sin^2 \dfrac{\theta}{2}.\] Lorsqu’on rencontre un groupement \(\sqrt{1+\cos\theta}\) ou \(\sqrt{1 - \cos\theta}\), il faut donc penser à l’angle moitié : \[\sqrt{1+\cos\theta} = \sqrt{2}\Bigl|\cos\dfrac{\theta}{2}\Bigr|, \quad \sqrt{1 - \cos\theta} = \sqrt{2}\Bigl|\sin\dfrac{\theta}{2}\Bigr|.\]

Comment retrouver les autres

Pour linéariser un produit de cosinus et de sinus, il suffit d’additionner ou de retrancher deux lignes des quatre formules fondamentales. Par exemple, si l’on veut retrouver la linéarisation de \(\cos a \cos b\), on part des deux premières lignes : \[\begin{cases} \cos(a+b) &= \cos a \cos b - \sin a \sin b \\ \cos(a-b) &= \cos a \cos b + \sin a \sin b \end{cases}\] et on fait \(L_2 + L_1\) pour obtenir \(\cos a \cos b = \dfrac{1}{2}\bigl[ \cos(a+b) + \cos(a-b)\bigr]\). On retrouve ainsi rapidement les trois formules suivantes :

\(\heartsuit\) \[\begin{aligned} \begin{alignedat}{2} \cos(a)\cos(b) &= \dfrac{1}{2}\bigl[ \cos(a+b) + \cos(a-b)\bigr] &\quad \mathop{\mathrm{ch}}(a)\mathop{\mathrm{ch}}(b) &= \dfrac{1}{2}\bigl[\mathop{\mathrm{ch}}(a+b) + \mathop{\mathrm{ch}}(a-b)\bigr] \\ \sin(a)\sin(b) &= \dfrac{1}{2}\bigl[ \cos(a-b) - \cos(a+b)\bigr] &\quad \mathop{\mathrm{sh}}(a)\mathop{\mathrm{sh}}(b) &= \dfrac{1}{2}\bigl[\mathop{\mathrm{ch}}(a+b)-\mathop{\mathrm{ch}}(a-b)\bigr]\\ \sin(a)\cos(b) &= \dfrac{1}{2}\bigl[\sin(a+b) + \sin(a-b) \bigr] &\quad \mathop{\mathrm{sh}}(a)\mathop{\mathrm{ch}}(b) &= \dfrac{1}{2}\bigl[\mathop{\mathrm{sh}}(a+b) + \mathop{\mathrm{sh}}(a-b)\bigr]. \end{alignedat} \end{aligned}\]

On sait également transformer une somme de cosinus ou une somme de sinus en un produit. Par exemple, pour exprimer \(\cos p + \cos q\), on part des deux premières lignes \[\begin{cases} \cos(a+b) &= \cos a \cos b - \sin a \sin b \\ \cos(a-b) &= \cos a \cos b + \sin a \sin b \end{cases}\] et, en les sommant, \[\cos(a+b) + \cos(a-b) = 2\cos(a)\cos(b).\] Il suffit de choisir \(a\) et \(b\) pour que \(\begin{cases} a + b &= p \\ a - b &= q \end{cases} \textrm{ i. e. } \begin{cases} a &= \dfrac{p+q}{2} \\ b &= \dfrac{p-q}{2} \end{cases}\). On obtient de cette façon les deux formules suivantes :

\(\heartsuit\) \[\begin{aligned} \begin{alignedat}{2} \cos p + \cos q &= 2\cos\bigl(\dfrac{p+q}{2}\bigr)\cos\bigl(\dfrac{p-q}{2} \bigr) & \quad \mathop{\mathrm{ch}}p + \mathop{\mathrm{ch}}q &= 2\mathop{\mathrm{ch}}\bigl(\dfrac{p+q}{2}\bigr)\mathop{\mathrm{ch}}\bigl(\dfrac{p-q}{2}\bigr) \newline \sin p + \sin q &= 2\sin\bigl(\dfrac{p+q}{2}\bigr)\cos\bigl(\dfrac{p-q}{2} \bigr) & \quad \mathop{\mathrm{sh}}p + \mathop{\mathrm{sh}}q &= 2\mathop{\mathrm{sh}}\bigl(\dfrac{p+q}{2}\bigr)\mathop{\mathrm{ch}}\bigl(\dfrac{p-q}{2}\bigr). \end{alignedat} \end{aligned}\]

On ne sait pas transformer \(\cos p + \sin q\) en un produit simple, sauf si \(p = q\) : \[\cos p + \sin p = \sqrt{2} \left(\cos(\pi / 4) \cos(p) + \sin(\pi / 4) \sin(p) \right)= \sqrt{2}\cos(p - \pi / 4)\] Plus généralement, \[a \cos \theta + b \sin \theta = \sqrt{a^2 + b^2}\Bigl(\underbrace{\dfrac{a}{\sqrt{a^2 + b^2}}}_{A}\cos \theta + \underbrace{\dfrac{b}{\sqrt{a^2 + b^2}}}_{B}\sin \theta\Bigr).\] Mais puisque \(A^2 + B^2 = 1\), il existe \(\varphi\in [0, 2\pi[\) tel que \(A = \cos\varphi\) et \(B = \sin \varphi\) d’où finalement \[a\cos \theta + b \sin \theta = \sqrt{a^2 + b^2} \bigl(\cos \theta \cos \varphi+ \sin \varphi\sin \theta) = \sqrt{a^2 + b^2} \cos(\theta - \varphi).\]

On retient le lien simple qui existe entre \(\tan x\) et \(\cos x\) :

\(\heartsuit\) \[\begin{aligned} 1 + \tan^2 x = \dfrac{1}{\cos^2 x}. \end{aligned}\]

Pour les tangentes, on a les formules suivantes qui proviennent de la définition et des formules fondamentales.

\(\heartsuit\) \[\begin{aligned} \begin{alignedat}{2} 1 + \tan^2 x &= \dfrac{1}{\cos^2 x} &\qquad 1 - \mathop{\mathrm{th}}^2 x &= \dfrac{1}{\mathop{\mathrm{ch}}^2 x} \\ \tan(a+b) &= \dfrac{\tan a + \tan b}{1 - \tan a \tan b} & \qquad \mathop{\mathrm{th}}(a+b) &= \dfrac{\mathop{\mathrm{th}}a + \mathop{\mathrm{th}}b}{1 + \mathop{\mathrm{th}}a \mathop{\mathrm{th}}b} \\ \tan(a-b) &= \dfrac{\tan a - \tan b}{1 + \tan a \tan b} & \qquad \mathop{\mathrm{th}}(a-b) &= \dfrac{\mathop{\mathrm{th}}a - \mathop{\mathrm{th}}b}{1 - \mathop{\mathrm{th}}a \mathop{\mathrm{th}}b} \\ \tan(2a) &= \dfrac{2\tan a}{1 - \tan^2 a} &\qquad \mathop{\mathrm{th}}(2a) &= \dfrac{2\mathop{\mathrm{th}}a}{1 + \mathop{\mathrm{th}}^2 a} . \end{alignedat} \end{aligned}\]

Terminons par des formules qui permettent d’exprimer les fonctions trigonométriques comme fractions rationnelles en \(t = \tan(x/2)\). Ces formules nous serviront pour calculer des primitives.

\(\heartsuit\) \[\begin{aligned} \begin{alignedat}{2} t&=\tan(\dfrac{x}{2}) &\qquad t&=\mathop{\mathrm{th}}(\dfrac{x}{2})\\ \sin x &= \dfrac{2t}{1+t^2} &\qquad \mathop{\mathrm{sh}}x & =\dfrac{2t}{1-t^2} \\ \cos x &= \dfrac{1-t^2}{1+t^2}&\qquad \mathop{\mathrm{ch}}x &=\dfrac{1+t^2}{1-t^2}\newline \tan x &= \dfrac{2t}{1-t^2} & \qquad \mathop{\mathrm{th}}x &=\dfrac{2t}{1+t^2}. \end{alignedat} \end{aligned}\]

Utilisons la formule \(\sin(x) = 2\sin(x/2)\cos(x/2)\) et la relation entre \(\cos\) et \(\tan\) : \(\cos^2(x/2) = \dfrac{1}{1+\tan^2(x/2)}\) pour écrire : \[\sin(x) = 2\tan(x/2)\cos^2(x/2) = \dfrac{2 t}{1+t^2}.\] De même, \[\cos(x) = 2\cos^2(x/2) - 1 = \dfrac{2}{1+t^2} - 1 = \dfrac{1-t^2}{1+t^2}\] Il suffit de former le quotient pour trouver \(\tan(x) = \dfrac{\sin x}{\cos x} = \dfrac{2t}{1-t^2}\).

Calculs de sommes

Comprendre les notations

On considère un ensemble \(I\) fini (indices) et une famille de réels \((a_i)_{i \in I}\). On note la somme de cette famille : \(\displaystyle{\sum_{i\in I}^{ }} a_i\) et le produit de cette famille : \(\displaystyle{\prod_{i\in I}^{ }} a_i\).

Par exemple, si \(I = [\kern-0.127em[ 1, n ]\kern-0.127em]\), \(\displaystyle{\sum_{i\in I}^{ }} a_i = a_1 + a_2 + \dots + a_n\) et \(\displaystyle{\prod_{i \in I}^{ }} a_i = a_1 \times a_2 \times \dots \times a_n\). Lorsque l’ensemble d’indices est un intervalle d’entiers : \(I = [\kern-0.127em[ p, q ]\kern-0.127em]\), on note \[\displaystyle{\sum_{i=p}^{q}} a_i = \displaystyle{\sum_{i\in I}^{ }} a_i = a_{p} + a_{p+1} + \dots + a_{q}.\]

Il est important de comprendre que la lettre \(i\) utilisée dans la notation \(\displaystyle{\sum_{i \in I}^{ }}\) est muette (elle prend les différentes valeurs dans \(I\)) et on peut la remplacer par une autre lettre : \[\displaystyle{\sum_{i=p}^{q}} a_i = \displaystyle{\sum_{j=p}^{q}} a_j = a_p + \dots + a_q.\] On sait calculer certaines sommes fondamentales, comme la somme des \(n\) premiers entiers (ainsi que la somme de leurs carrés et de leurs cubes) : \(\heartsuit\) \[\begin{aligned} \displaystyle{\sum_{i=1}^{n}} i = \dfrac{n(n+1)}{2} \end{aligned}\]

\(\heartsuit\) \[\begin{aligned} \displaystyle{\sum_{i=1}^{n}} i^2 = \dfrac{n(n+1)(2n+1)}{6} \end{aligned}\]

\(\heartsuit\) \[\begin{aligned} \displaystyle{\sum_{i=1}^{n}} i^3 = \dfrac{n^2(n+1)^2}{4} = \left(\displaystyle{\sum_{i=1}^{n}} i\right)^2. \end{aligned}\]

Pour la somme \(S\) des \(n\) premiers entiers, une démonstration simple consiste à écrire les termes en ordre inverse : \[\begin{cases} S &= 1 + 2 + \dots + (n-1) + n \\ S &= n + (n-1) + \dots + 2 + 1 \end{cases}\] et, en remarquant que \(1+n = 2 + (n-1) = \dots = n + 1\) puis en additionnant ces deux lignes, on obtient \(2S = n(n+1)\) d’où le résultat. On montre les deux autres formules par récurrence sur \(n\).

La formule du binôme de Newton : \[(a+b)^n = \dbinom{n}{0}a^n + \dbinom{n}{1} a^{n-1}b + \dbinom{n}{2} a^{n-2}b^2 + \dots + \dbinom{n}{n} b^n\] s’écrit avec nos notations : \(\heartsuit\) \[\begin{aligned} (a+b)^n = \displaystyle{\sum_{k=0}^{n}} \dbinom{n}{k} a^{n-k} b^{k}. \end{aligned}\]

Les sommes géométriques sont très importantes. Si \(x \neq 1\), \[S = 1 + x + x^2 + \dots + x^n = \dfrac{1-x^{n+1}}{1-x}.\] En effet, \[\begin{cases} S &= 1 + x + \dots + x^n \\ xS &= x + \dots + x^n + x^{n+1} \end{cases}\] et, en soustrayant, on obtient \((1-x) S = 1 - x^{n+1}\) d’où le résultat. Lorsque \(x = 1\), on somme \(1\) \((n+1)\) fois : \(S = (n+1)\). Cette formule s’écrit avec nos notations : \(\heartsuit\) \[\begin{aligned} \displaystyle{\sum_{i=0}^{n}} x^i = \begin{cases} \dfrac{1-x^{n+1}}{1-x} & \textrm{ si } x \neq 1 \newline (n+1) & \textrm{ si } x = 1 \end{cases}. \end{aligned}\]

Changement d’indices, télescopage

Considérons deux entiers \(n<p\) et une somme \[S = \displaystyle{\sum_{k=n}^{p}} a_k = a_n + a_{n+1} + \dots + a_p.\] On peut également écrire \[S = \displaystyle{\sum_{i=0}^{p-n}} a_{n+i} = a_n + a_{n+1} + \dots + a_{n + (p-n)}.\] Formellement, on effectue un changement d’indice de sommation en posant \(i = k - n\) et en modifiant les bornes de l’intervalle de sommation : lorsque \(k=n\), \(i=0\) et, lorsque \(k=p\), \(i=p-n\). Dans le terme générique à sommer \(a_k\), on remplace toutes les occurrences de \(k\) par \((n+i)\).

On peut également effectuer le changement d’indices \(j = p-k\) : lorsque \(k\) parcourt l’intervalle \([\kern-0.127em[ n, p ]\kern-0.127em]\), \(j\) parcourt l’intervalle \([\kern-0.127em[ 0, p-n ]\kern-0.127em]\). En remplaçant \(k\) par \(p-j\), on obtient alors : \[S = \displaystyle{\sum_{j=0}^{p-n}} a_{p-j} = a_p + a_{p-1} + \dots + \underbrace{a_{p - (p-n)}}_{a_n}.\] Ce changement d’indice consiste à sommer les termes de la famille dans l’ordre inverse. La formule du binôme peut s’écrire : \[(a+b)^n = \displaystyle{\sum_{k=0}^{n}} \dbinom{n}{k} a^kb^{n-k} = \displaystyle{\sum_{j=0}^{n}} \dbinom{n}{j} a^{n-j}b^j.\] Il suffit d’échanger les rôles de \(a\) et \(b\) ou d’effectuer le changement d’indice \(j = n-k\).

Si dans les termes à sommer, on a en facteur un terme qui ne dépend pas de l’indice de sommation, on peut le factoriser dans la somme : \[S = \displaystyle{\sum_{i=1}^{n}} \alpha \times a_i = (\alpha a_1) + (\alpha a_2) + \dots + (\alpha a_n) = \alpha(a_1+\dots+a_n) = \alpha \displaystyle{\sum_{i=1}^{n}} a_i.\]

Par exemple, pour calculer une somme géométrique avec l’indice qui ne commence pas à \(0\), on peut utiliser deux techniques : \[S = \displaystyle{\sum_{i=p}^{q}} x^i = \displaystyle{\sum_{i=0}^{q}} x^i - \displaystyle{\sum_{i=0}^{p-1}} x^i = \dfrac{1-x^{q+1}}{1-x} - \dfrac{1-x^{p}}{1-x} = \dfrac{x^p - x^{q+1}}{1-x}\] ou, en posant \(j=i-p\), \[S = \displaystyle{\sum_{i=p}^{q}} x^i = \displaystyle{\sum_{j=0}^{q-p}} x^{j+p} %\quad (j = i - p) = \displaystyle{\sum_{j=0}^{q-p}} x^p x^j = x^p \displaystyle{\sum_{j=0}^{q-p}} x^p = x^p \dfrac{1-x^{q-p+1}}{1-x}.\] En pratique, il vaut mieux utiliser la deuxième méthode car elle donne un résultat factorisé dans des calculs de sommes plus compliqués.

Pour calculer la somme \(S=\displaystyle{\sum_{i=0}^{n}} (n-i)^2\), on peut développer : \[\begin{aligned} S &= \displaystyle{\sum_{i=0}^{n}} n^2 - \displaystyle{\sum_{i=0}^{n}} 2ni + \displaystyle{\sum_{i=0}^{n}} i^2 \\ &= (n+1)n^2 - 2n \displaystyle{\sum_{i=0}^{n}} i + \displaystyle{\sum_{i=0}^{n}} i^2 \\ &= (n+1)n^2 - 2n \dfrac{n(n+1)}{2} + \dfrac{n(n+1)(2n+1)}{6}. \end{aligned}\] Mais il vaut bien mieux effectuer le changement d’indices \(k = n-i\) : \[S = \displaystyle{\sum_{k=0}^{n}} k^2 = \dfrac{n(n+1)(2n+1)}{6}.\]

Voyons maintenant une technique courante dans les calculs de sommes, le télescopage. Considérons : \[S = \displaystyle{\sum_{k=1}^{n}} \Bigl(\dfrac{1}{k+1} - \dfrac{1}{k}\Bigr).\] Si l’on écrit les termes successifs de cette somme, \[S = \Bigl(\dfrac{1}{2} - \dfrac{1}{1}\Bigr) + \Bigl(\dfrac{1}{3} - \dfrac{1}{2}\Bigr) + \dots + \Bigl(\dfrac{1}{n+1} - \dfrac{1}{n}\Bigr),\] on s’aperçoit que tous les termes se simplifient sauf \(1/1\) et \(1/(n+1)\) : \(S = \dfrac{1}{n+1}-1\). Utilisons les changements d’indices pour rédiger ce calcul : \[\begin{aligned} S &= \displaystyle{\sum_{k=1}^{n}} \dfrac{1}{k+1} - \displaystyle{\sum_{k=1}^{n}} \dfrac{1}{k} \\ &= \displaystyle{\sum_{i=1}^{n+1}} \dfrac{1}{i} - \displaystyle{\sum_{k=1}^{n}} \dfrac{1}{k} \quad (i = k + 1) \\ &= \displaystyle{\sum_{i=1}^{n+1}} \dfrac{1}{i} - \displaystyle{\sum_{i=1}^{n}} \dfrac{1}{i} \quad (\textrm{ les indices sont muets }) \\ &= \displaystyle{\sum_{i=2}^{n}} \dfrac{1}{i} + \dfrac{1}{n+1} - \Bigl(\displaystyle{\sum_{i=2}^{n}} \dfrac{1}{i} + \dfrac{1}{1}\Bigr) \textrm{ (on sort des sommes les termes qui ne sont pas communs)} \newline &= \dfrac{1}{n+1} - 1.\end{aligned}\]

Calculer la limite de la suite de terme général \[S_n = \displaystyle{\sum_{k=1}^{n}} \dfrac{1}{k(k+1)(k+2)}\] Commençons par décomposer en éléments simples la fraction rationnelle \[\dfrac{1}{k(k+1)(k+2)} = \dfrac{1/2}{k} - \dfrac{1}{k+1} + \dfrac{1/2}{k+2}.\] pour écrire \[\begin{aligned} S &= \dfrac{1}{2} \displaystyle{\sum_{k=1}^{n}} \dfrac{1}{k} - \displaystyle{\sum_{k=1}^{n}} \dfrac{1}{k+1} + \dfrac{1}{2} \displaystyle{\sum_{k=1}^{n}} \dfrac{1}{k+2} \\ &= \dfrac{1}{2} \displaystyle{\sum_{k=1}^{n}} \dfrac{1}{k} - \displaystyle{\sum_{i=2}^{n+1}} \dfrac{1}{i} + \dfrac{1}{2} \displaystyle{\sum_{j=3}^{n+2}} \dfrac{1}{j} \\ &= \dfrac{1}{2}\Bigl( \displaystyle{\sum_{k=3}^{n}} \dfrac{1}{k} + \dfrac{1}{1} + \dfrac{1}{2}\Bigr) - \Bigl(\displaystyle{\sum_{k=3}^{n}} \dfrac{1}{k} + \dfrac{1}{2} + \dfrac{1}{n+1}\Bigr) + \dfrac{1}{2}\Bigl(\displaystyle{\sum_{k=3}^{n}} \dfrac{1}{k} + \dfrac{1}{n+1} + \dfrac{1}{n+2}\Bigr) \\ &= \dfrac{1}{4} -\dfrac{1}{2(n+1)} + \dfrac{1}{2(n+2)}\xrightarrow[n \rightarrow +\infty]{}\dfrac{1}{4}. \end{aligned}\]

On utilise souvent des changements d’indice pour calculer des sommes où tous les indices sont pairs ou impairs. Par exemple, on veut calculer la somme des premiers entiers pairs inférieurs à un entier \(n\) : \[S_n = 0 + 2 + 4 + \dots = \displaystyle{\sum_{i=0 \atop i pair}^{n}} i.\] Puisque les indices \(i\) de cette somme sont pairs, on peut les écrire sous la forme \(i = 2k\)\(k\) est un entier. Comme \(0 \leqslant i =2k \leqslant n\), il vient \(0 \leqslant k \leqslant n /2\) et le nouvel indice \(k\) varie donc dans l’intervalle entier \([\kern-0.127em[ 0, E(n/2) ]\kern-0.127em]\). Si \(n = 2p\), \(k \in [\kern-0.127em[ 0, p ]\kern-0.127em]\) et si \(n = 2p+1\), \(k \in [\kern-0.127em[ 0, p ]\kern-0.127em]\). Par conséquent, \[S_n = \displaystyle{\sum_{k=0}^{p}} (2k) = 2 \displaystyle{\sum_{k=0}^{p}} k = 2 \dfrac{p(p+1)}{2} = p(p+1).\]

Nous voulons calculer maintenant la somme des coefficients binomiaux \[S_n = \dbinom{n}{0} + \dbinom{n}{2} + \dots = \displaystyle{\sum_{i=0 \atop i pair }^{n}} \dbinom{n}{i}.\] L’idée consiste à utiliser la formule du binôme donnant la somme de tous les coefficients binomiaux : \[2^n = (1+1)^n = \dbinom{n}{0} + \dbinom{n}{1} + \dbinom{n}{2} + \dots = \displaystyle{\sum_{i=0}^{n}} \dbinom{n}{i} .\] On introduit la somme des coefficients binomiaux où les indices sont impairs : \[T_n = \displaystyle{\sum_{i=0 \atop i\ impair }^{n}} \dbinom{n}{i}.\] Nous avons alors \(S_n + T_n = 2^n\). Il nous faut une autre relation pour calculer \(S_n\) et \(T_n\). Formons la différence : \[S_n - T_n = \dbinom{n}{0} - \dbinom{n}{1} + \dbinom{n}{2} - \dbinom{n}{3} +\dots = \displaystyle{\sum_{i=0}^{n}}(-1)^i \dbinom{n}{i} = (1-1)^n = 0.\] On en déduit que \(S_n = T_n = 2^{n-1}\). Nous avons calculé les deux sommes en développant \((1+1)^n\) et \((1-1)^n\). On remarque que \(1\) et \(-1\) sont les racines carrées de l’unité.

On peut généraliser cette idée en utilisant les racines \(n\)-ièmes de l’unité. Par exemple, si nous voulons calculer les sommes \[\begin{cases} U_n &= \displaystyle{\sum_{k=0 \atop k \equiv 0 \; [3]}^{n}} \dbinom{n}{k}, \\ V_n &= \displaystyle{\sum_{k=0 \atop k \equiv 1 \; [3]}^{n}} \dbinom{n}{k}, \\ W_n &= \displaystyle{\sum_{k=0 \atop k \equiv 2 \; [3]}^{n}} \dbinom{n}{k}, \end{cases}\] on calcule à l’aide du binôme les trois sommes \[\begin{cases} (1+1)^n &= \displaystyle{\sum_{k=0}^{n}} \dbinom{n}{k} = U_n + V_n + W_n,\\ (1+j)^n &= \displaystyle{\sum_{k=0}^{n}} \dbinom{n}{k} j^k = U_n + jV_n + j^2 W_n,\\ (1+j^2)^n&= \displaystyle{\sum_{k=0}^{n}} \dbinom{n}{k} j^{2k} = U_n + j^2V_n + j W_n. \end{cases}\] Nous avons utilisé les relations \(j^k =1\) lorsque \(k \equiv 0 \; [3]\), \(j^k = j\) lorsque \(k \equiv 1 \; [3]\) et \(j^k = j^2\) lorsque \(j \equiv 2 \; [3]\). En utilisant la propriété \(1 + j + j^2 = 0\), il suffit d’additionner les trois lignes précédentes pour trouver \[U_n = \dfrac{2^n + (1+j)^n + (1+j^2)^n}{3}.\] On peut également avec la combinaison linéaire \(L_1 + j^2 L_2 +jL_3\) calculer \(V_n\) et de même avec \(L_1 + jL_2 + j^2L_3\) calculer \(W_n\). Cette technique permet de calculer plus généralement des sommes de la forme \[\displaystyle{\sum_{k=0 \atop k \equiv l \; [p]}^{n}} \dbinom{n}{k}.\] Il suffit de développer \((1+\omega_k)^n\) pour les \(n\) racines \(n\)-ièmes de l’unité \(\omega_k\).

Sommes doubles

Lorsque l’ensemble d’indices est une partie finie formée de couples, on dit qu’on a une somme double. Par exemple, si \(I = \{(1,2), (1,3), (2,2), (3,2)\}\) et \(a_{(i,j)} = i+j\), \[S = \displaystyle{\sum_{i \in I}^{ }} a_i = a_{(1,2)} + a_{(3,1)} + a_{(2,1)} + a_{(1,1)} = (1+2) + (1+3) + (2+2) + (3+2) = 16.\] Un cas courant est lorsque \(I\) est un produit d’intervalles d’entiers : \[I = [\kern-0.127em[ 1, p ]\kern-0.127em] \times [\kern-0.127em[ 1, q ]\kern-0.127em] = \{(1,1),(1,2),\dots, (1,q), (2,1)\dots, (2,q), \dots, (p,q)\}\] En notant \(a_{ij} = a_{(i,j)}\), on a la formule d’interversion de sommes : \[\displaystyle{\sum_{(i,j) \in I \times J}^{ }} a_{ij} = \displaystyle{\sum_{i=1}^{p}} \underbrace{\Bigl(\displaystyle{\sum_{j=1}^{q}} a_{ij}\Bigr)}_{\alpha_i} = \displaystyle{\sum_{j=1}^{q}} \underbrace{\Bigl( \displaystyle{\sum_{i=1}^{p}} a_{ij}\Bigr)}_{\beta_j}.\] Remarquons que \(\alpha_i = \displaystyle{\sum_{j=1}^{q}} a_{ij}\) ne dépend pas de \(j\) (l’indice de sommation est muet), mais de \(i\). De même, \(\beta_j\) ne dépend pas de \(i\). La formule précédente permet donc d’exprimer une somme double à l’aide de sommes simples.

Pour comprendre cette formule, considérons l’exemple suivant : \(I = \{1,2\}\) et \(J = \{1,2,3\}\) : \[\displaystyle{\sum_{i=1}^{2}} \Bigl(\displaystyle{\sum_{j=1}^{3}} a_{ij}\Bigr) = \underbrace{(a_{11} + a_{12} + a_{13})}_{i=1} + \underbrace{(a_{21} + a_{22} + a_{23})}_{i=2},\] \[\displaystyle{\sum_{j=1}^{3}} \Bigl(\displaystyle{\sum_{i=1}^{2}} a_{ij}\Bigr) = \underbrace{(a_{11} + a_{21})}_{j=1} + \underbrace{(a_{12} + a_{22})}_{j=2} + \underbrace{(a_{13} + a_{23})}_{j=3}.\] Ces deux sommes sont bien égales puisque le résultat d’une sommation ne dépend pas de l’ordre dans lequel on effectue les sommes (en algèbre, on dit que la loi \(+\) est commutative).

Lorsque \(a_{ij} = \alpha_i \times \beta_j\), la somme double s’écrit comme un produit de deux sommes simples : \[\begin{aligned} S &= \displaystyle{\sum_{i=1}^{p}} \displaystyle{\sum_{j=1}^{q}} \alpha_i \beta_j \\ &= \displaystyle{\sum_{i=1}^{p}} \alpha_i \underbrace{\Bigl(\displaystyle{\sum_{j=1}^{q}} \beta_j\Bigr)}_{\gamma} \quad (\alpha_i \textrm{ ne dépend pas de } j) \newline &= \Bigl(\displaystyle{\sum_{j=1}^{q}} \beta_j\Bigr) \times \Bigl(\displaystyle{\sum_{i=1}^{p}} \alpha_i\Bigr) \quad (\gamma \textrm{ ne dépend pas de } i).\end{aligned}\]

Calculons la somme \(S = \displaystyle{\sum_{j=0}^{n}} \displaystyle{\sum_{i=0}^{p}} i\dbinom{n}{j} (\sqrt{2})^j\) : \[\begin{aligned} S &= \Bigl(\displaystyle{\sum_{i=0}^{p}} i\Bigr)\Bigl(\displaystyle{\sum_{j=0}^{n}} \dbinom{n}{j} \sqrt{2}^j\Bigr) \newline &= \dfrac{p(p+1)}{2} (1+\sqrt{2})^n. \end{aligned}\]
Calculons la somme \(S = \displaystyle{\sum_{i=1}^{n}} \displaystyle{\sum_{j=1}^{n}} (i+j)^2\). En développant cette somme, \[\begin{aligned} S &= \displaystyle{\sum_{i=1}^{n}}\displaystyle{\sum_{j=1}^{n}} i^2 + 2\displaystyle{\sum_{i=1}^{n}} \displaystyle{\sum_{j=1}^{n}} ij + \displaystyle{\sum_{i=1}^{n}} \displaystyle{\sum_{j=1}^{n}} j^2 \\ &= n \displaystyle{\sum_{i=1}^{n}} i^2 + 2\Bigl(\displaystyle{\sum_{i=1}^{n}} i\Bigr)\Bigl(\displaystyle{\sum_{j=1}^{n}} j\Bigr) + n\displaystyle{\sum_{j=1}^{n}} j^2 \\ &= 2n\displaystyle{\sum_{i=1}^{n}} i^2 + 2\bigl(\displaystyle{\sum_{i=1}^{n}} i\Bigr)^2 \quad (\textrm{ indices muets }) \\ &= \dfrac{n^2(n+1)(7n+5)}{6}. \end{aligned}\]

Un exemple plus compliqué d’interversion de signes sommes, lorsque les bornes ne sont pas indépendantes : \[S = \displaystyle{\sum_{i=0}^{n}} \Bigl(\displaystyle{\sum_{j=0}^{i}} a_{ij}\Bigr) = \displaystyle{\sum_{j=0}^{n}} \Bigl(\displaystyle{\sum_{i=j}^{n}} a_{ij} \Bigr).\] Ici, \(S = \sum_{(i, j) \in I} a_{ij}\)\(I = \{(i,j) \in \mathbb N[2] \mid 0 \leqslant j \leqslant i \leqslant n\}\). On peut représenter chaque couple \((i,j) \in I\) par un point sur un quadrillage ( \(n=3\) sur la figure) :

Si on commence par sommer sur l’indice \(i\) variant de \(0\) à \(n\), à \(i\) fixé, \(j\) varie entre \(0\) et \(i\). Sur le dessin, cela correspond à sommer verticales par verticales : \[S = \underbrace{(a_{00})}_{i=0} + \underbrace{(a_{10} + a_{11})}_{i=1} + \underbrace{(a_{20} + a_{21} + a_{22})}_{i=2} + \underbrace{(a_{30} + a_{31} + a_{32} + a_{33})}_{i=3}.\] Mais on peut également sommer horizontale par horizontale : \[S = \underbrace{(a_{00} + a_{10} + a_{20} + a_{30})}_{j=0} + \underbrace{(a_{11} + a_{21} + a_{31})}_{j=1} + \underbrace{(a_{22} + a_{32})}_{j=2}+ \underbrace{(a_{33})}_{j=3}.\]

Calculer la somme \(S = \displaystyle{\sum_{i=0}^{n}} \displaystyle{\sum_{j=i}^{n}} \dbinom{n}{j} \dbinom{j}{i}\). En inversant les signes sommes : \[S = \displaystyle{\sum_{j=0}^{n}} \displaystyle{\sum_{i=0}^{j}} \dbinom{n}{j} \dbinom{j}{i} = \displaystyle{\sum_{j=0}^{n}} \dbinom{n}{j} \displaystyle{\sum_{i=0}^{j}} \dbinom{j}{i} = \displaystyle{\sum_{j=0}^{n}} \dbinom{n}{j} 2^j = (1+2)^n = 3^n\]

Trigonométrie et nombres complexes

Nous allons voir dans ce paragraphe des techniques de calcul trigonométrique utilisant la notation \(\sum\) et les exponentielles imaginaires.

Transformation de \(\cos(n\theta)\)

Il s’agit de transformer une combinaison linéaire de \(\cos k\theta\) et \(\sin k\theta\) en produits de la forme \(\cos^p a \sin ^q a\). Ce procédé s’avérera utile pour résoudre des équations trigonométriques. La formule de Moivre est à la base du procédé de factorisation. Commençons par un exemple. Pour \(\theta\in\mathbb{R}\), on veut transformer en produits \(\cos 5 \theta\) et \(\sin 5 \theta\). On a, par application de la formule de Moivre : \[\bigl(\cos \theta + i \sin \theta\bigr)^5= e^{5i\theta} = \cos 5\theta + i \sin 5\theta .\] Développons le membre de gauche de cette égalité avec la formule du binôme : \[\begin{aligned} {\left(\cos \theta + i \sin \theta\right)}^5 &=&\cos^5 \theta + 5 i \cos^4 \theta \sin \theta - 10 \cos^3 \theta\sin^2 \theta - 10 i \cos^2 \theta \sin^3 \theta + 5 \cos\theta \sin^4\theta + i \sin^5 \theta\\ &=&\left(\cos^5 \theta - 10 \cos^3 \theta\sin^2 \theta + 5 \cos\theta \sin^4\theta\right) + i\left( 5\cos^4 \theta \sin \theta - 10 \cos^2 \theta \sin^3 \theta + \sin^5 \theta\right) . \end{aligned}\] En prenant la partie réelle et imaginaire, on obtient : \[\begin{aligned} \cos 5\theta&=&\cos^5 \theta - 10 \cos^3 \theta\sin^2 \theta + 5 \cos\theta \sin^4\theta\\ \sin 5\theta&=&5\cos^4 \theta \sin \theta - 10 \cos^2 \theta \sin^3 \theta + \sin^5 \theta .\end{aligned}\] Calculons maintenant une formule générale qui permet d’exprimer \(\cos(n\theta)\) comme un polynôme en \(\cos(\theta)\). Écrivons avec la formule de Moivre \[\cos(n\theta) = \dfrac{1}{2}\bigl(e^{in\theta} + e^{-in\theta}\bigr) = \dfrac{1}{2}\Bigl[(\cos\theta + i\sin\theta)^n + (\cos\theta -i \sin\theta)^n\Bigr]\] et développons à l’aide de la formule du binôme : \[\cos(n\theta) = \dfrac{1}{2}\displaystyle{\sum_{k=0}^{n}} \dbinom{n}{k} \bigl[1 + (-1)^k\bigr] i^k \cos^{n-k}\theta \sin^k\theta.\] Puisque \([1 + (-1)^k] = \begin{cases} 2 & \textrm{ si } k \textrm{ pair } \\ 0 & \textrm{ si } k \textrm{ impair } \end{cases}\), \(\cos(n\theta) = \displaystyle{\sum_{k=0 \atop k pair}^{n}} \dbinom{n}{k} i^k\cos^{n-k}\theta \sin^k\theta\). Tous les indices de cette somme étant pairs, on peut les écrire \(k = 2p\) avec \(0 \leqslant 2p \leqslant n\), c’est-à-dire \(p \in [\kern-0.127em[ 0, E(n/2) ]\kern-0.127em]\). Par conséquent, \[\cos(n\theta) = \displaystyle{\sum_{p=0}^{E(n/2)}} \dbinom{n}{2p} (-1)^p\cos^{n-2p}\theta \sin^{2p}\theta = \displaystyle{\sum_{p=0}^{E(n/2)}} \dbinom{n}{2p} \cos^{n-2p}\theta (1-\cos^2\theta)^p = T_n(\cos\theta)\]\(T_n\) est un polynôme appelé \(n\)-ième polynôme de Tchebychev (de première espèce) : \[T_n(X) = \displaystyle{\sum_{p=0}^{E(n/2)}} \dbinom{n}{2p} (-1)^p X^{n-2p} (1-X^2)^p.\] Le calcul précédent est fondamental. Les polynômes de Tchebychev jouent un rôle important en algèbre et en analyse et il est courant de trouver cette question de cours dans les premières questions d’un problème de concours. Vous devez avoir bien compris et savoir refaire rapidement ce calcul.

Effectuons le calcul similaire pour \(\sin(n\theta)\) : \[\sin(n\theta) = \dfrac{1}{2i}\Bigl(e^{in\theta} - e^{-in\theta}\Bigr) = \dfrac{1}{2i} \Bigl[ (\cos\theta + i\sin\theta)^n - (\cos\theta - i\sin\theta)^n \Bigr],\] \[\sin(n\theta) = \dfrac{1}{2i} \displaystyle{\sum_{k=0}^{n}} \dbinom{n}{k} \bigl[1 - (-1)^k\bigr] i^k \cos^{n-k}\theta\sin^{k}\theta = \dfrac{1}{i} \displaystyle{\sum_{k=0 \atop k impair}^{n}} \dbinom{n}{k} i^k \cos^{n-k}\theta \sin^k\theta.\] Tous les indices de cette somme étant impairs, on peut les écrire sous la forme \(k = 2p+1\)\(0 \leqslant 2p+1 \leqslant n\), c’est-à-dire \(-1/2 \leqslant p \leqslant(n-1)/2\) et \(p\) varie donc dans l’intervalle d’entiers \([\kern-0.127em[ 0, E({\scriptstyle n-2\over\scriptstyle 2}) ]\kern-0.127em]\). \[\sin(n\theta) = \dfrac{1}{i}\displaystyle{\sum_{p=0}^{E({\scriptstyle n-1\over\scriptstyle 2})}} \dbinom{n}{2p+1} i^{2p+1} \cos^{n-2p-1}\theta \sin^{2p+1}\theta = \sin\theta \displaystyle{\sum_{p=0}^{E({\scriptstyle n-1\over\scriptstyle 2})}} \dbinom{n}{2p+1} (-1)^p \cos^{n-2p-1}\theta(1-\cos^2\theta)^{p} = \sin\theta U_n(\cos\theta)\]\(U_n(X) = \displaystyle{\sum_{p=0}^{E({\scriptstyle n-1\over\scriptstyle 2})}} \dbinom{n}{2p+1}(-1)^p X^{n-2p-1} (1-X^2)^p\) est un polynôme.

Pour terminer, voyons un complément important sur les polynômes de Tchebychev. Utilisons la formule de trigonométrie \[\cos p + \cos q = 2 \cos\bigl(\dfrac{p+q}{2}\bigr)\cos\bigl(\dfrac{p-q}{2}\bigr)\] pour écrire \(\forall \theta \in \mathbb{R}\), \(\forall n \geqslant 2\), \[\cos\bigl[(n+1)\theta\bigr] + \cos\bigl[(n-1)\theta\bigr] = 2\cos\theta \cos(n\theta).\] Puisque tout réel \(x \in [-1,1]\) peut s’écrire \(\cos(\theta)\), il vient \[\forall x \in [-1, 1], \quad T_{n+1}(x) + T_{n-1}(x) = 2x T_n(x)\] et on utilise une technique importante sur les polynômes : le polynôme \(Q = T_{n+1}+T_{n-1} - 2XT_n\) admet une infinité de racines, il est donc nul. On obtient ainsi une relation de récurrence qui permet de calculer de proche en proche les \(T_n\) : \[T_0 = 1, T_1 = X, \forall n \geqslant 2, \quad T_{n+1} = 2X T_n - T_{n-1}.\] On obtient \(T_2(X) = 2X^2-1\), \(T_3 = 4X^3-3X\) et on retrouve les formules de trigonométrie \(\cos(2\theta) = 2\cos^2\theta - 1\), \(\cos(3\theta) = 4\cos^3\theta - 3\cos\theta\) et ainsi de suite. On se sert souvent de cette formule de récurrence pour introduire plus simplement les polynômes de Tchebychev. Elle permet en particulier de montrer simplement par récurrence que le terme dominant du polynôme \(T_n\) vaut \(2^{n-1} X^n\).

Problèmes de linéarisation

L’objectif de ce paragraphe est de montrer comment transformer un produit \(\cos^p \theta \sin ^q \theta\) en une combinaison linéaire de \(\cos k\theta\) et \(\sin k\theta\). Cette opération s’avérera très utile pour les problèmes de calcul de primitive. Les formules d’Euler sont également à la base du processus de linéarisation. Commençons par un exemple. Nous souhaitons linéariser \(\sin^6 \theta\). On a : \[\begin{aligned} \sin^6 \theta&=&\left(\dfrac{e^{i \theta}-e^{-i \theta}}{2i}\right)^6 \\ &=& -\dfrac{1}{64}\left(e^{i6\theta}-6e^{i5\theta}e^{-i\theta}+15e^{i4\theta}e^{-i2\theta}-20e^{i3\theta}e^{-i3\theta}+15e^{i2\theta}e^{-i4\theta}-6e^{i\theta}e^{-i5\theta}+ e^{-i6\theta} \right)\\ &=& -\dfrac{1}{64}\left(e^{i6\theta}-6e^{i4\theta}+15e^{i2\theta}-20+15e^{-i2\theta}-6e^{-i4\theta}+ e^{-i6\theta} \right) \\ &=& -\dfrac{1}{64}\left(e^{i6\theta}+ e^{-i6\theta}-6 \left(e^{i4\theta}+ 6e^{-i4\theta}\right) +15\left(e^{i2\theta}+e^{-i2\theta}\right) -20 \right) \\ &=& -\dfrac{1}{32}\left(\cos 6\theta-6\cos 4 \theta + 15\cos 2\theta-10\right).\end{aligned}\] Nous avons utilisé la formule du binôme et avons regroupé les termes symétriques pour retrouver des cosinus. Maple permet d’effectuer ces linéarisations trigonométriques à l’aide de la fonction combine :

 > A:=(cos(t))^4;
                                      4
                           A := cos(t) 
 > combine(A);
                   1            1            3
                   - cos(4 t) + - cos(2 t) + -
                   8            2            8

Il est alors facile de déterminer une primitive, \(\int \cos^4 t \mathrm{ \;d}t = \dfrac{1}{32} \sin(4t) + \dfrac{1}{4} \sin(2t) + \dfrac{3}{8} t + C\).

Attaquons-nous maintenant à la linéarisation générale : \[\cos^n \theta = \dfrac{1}{2^n}\Bigl(e^{i\theta} + e^{-i\theta}\Bigr)^n = \dfrac{1}{2^n}\displaystyle{\sum_{k=0}^{n}} \dbinom{n}{k} e^{i(n-2k)\theta}.\] En s’inspirant de l’exemple précédent, nous souhaitons regrouper le terme \(k=0\) avec le terme \(k=n\), le terme \(k=1\) avec le terme \(k= n-1\) ... Il faut distinguer les cas \(n\) pair et \(n\) impair pour pouvoir compter précisément le nombre de termes dans la somme.

  • Si \(n = 2p+1\) est impair, il y a \(2p+2\) termes dans la somme que l’on sépare en deux : \[2^n\cos^n\theta = \displaystyle{\sum_{k=0}^{p}} \dbinom{n}{k} e^{i(n-2k)\theta} + \displaystyle{\sum_{k=p+1}^{2p+1}} \dbinom{n}{k} e^{i(n-2k)\theta}.\] Effectuons alors un changement d’indice dans la deuxième somme pour inverser l’ordre des termes : \(k' = n-k\). On remarque que \(\dbinom{n}{n-k'} = \dbinom{n}{k'}\) et que \((n-2(n-k')) = -(n-2k')\) d’où \[2^n\cos^n\theta = \displaystyle{\sum_{k=0}^{p}} \dbinom{n}{k} e^{i(n-2k)\theta} + \displaystyle{\sum_{k'=0}^{p}} \dbinom{n}{k'} e^{-i(n-2k')} = \displaystyle{\sum_{k=0}^{p}} \dbinom{n}{k} \bigl[e^{i(n-2k)\theta} + e^{-i(n-2k)\theta}\bigr]\] et finalement, \[\cos^{2p+1}\theta = \dfrac{1}{2^{2p}} \displaystyle{\sum_{k=0}^{p}} \dbinom{2p+1}{k} \cos[(2p-2k+1)\theta].\]

  • Si \(n = 2p\) est pair, il y a \(2p+1\) termes dans la somme et on doit sortir le terme médian : les \((p-1)\) premiers termes correspondent à \(k \in [\kern-0.127em[ 0, p-1 ]\kern-0.127em]\), le terme médian à \(k=p\) et les \((p-1)\) derniers termes à \(k \in [\kern-0.127em[ p+1, 2p ]\kern-0.127em]\) : \[2^n \cos^n\theta = \displaystyle{\sum_{k=0}^{p-1}} \dbinom{n}{k} e^{i(n-2k)\theta} + \dbinom{2p}{p} + \displaystyle{\sum_{k=p+1}^{2p}} \dbinom{n}{k} e^{i(n-2k)\theta}\] et avec le changement d’indice \(k'=n-k\) dans la deuxième somme puis en regroupant les sommes, on trouve finalement \[\cos^{2p}\theta = \dfrac{\dbinom{2p}{p}}{2^{2p}} + \dfrac{1}{2^{2p-1}}\displaystyle{\sum_{k=0}^{p-1}} \dbinom{2p}{k} \cos\bigl(2(p-k)\theta\bigr).\]

Utilisation des sommes géométriques complexes

L’idée fondamentale de ce paragraphe est qu’il est souvent plus facile de calculer une somme complexe qu’une somme trigonométrique, grâce aux propriétés de l’exponentielle imaginaire.

Pour calculer la somme \[S_n = 1 + \cos(\theta) + \cos(2\theta) + \dots + \cos(n\theta) = \displaystyle{\sum_{k=0}^{n}} \cos(k\theta),\] introduisons la somme correspondante avec des sinus : \[T_n = 0 + \sin(\theta) + \sin(2\theta) + \dots + \sin(n\theta) = \displaystyle{\sum_{k=0}^{n}} \sin(k\theta)\] et la somme des exponentielles imaginaires associées : \[U_n = S_n + i T_n = \displaystyle{\sum_{k=0}^{n}} e^{ik\theta} = \displaystyle{\sum_{k=0}^{n}} \bigl[e^{i\theta}\bigr]^k.\] Si on sait calculer \(U_n\), on en déduit \(S_n\) et \(T_n\) en prenant les parties réelles et imaginaires : \(S_n = \mathop{\mathrm{Re}}(U_n)\) et \(T_n=\mathop{\mathrm{Im}}(U_n)\). On reconnaît pour \(U_n\) une somme géométrique de raison \(e^{i\theta}\). Distinguons deux cas :

  1. Si \(e^{i\theta} \neq 1\), c’est-à-dire \(\theta \neq 2k\pi\)\(k\in\mathbb{Z}\), alors \[U_n = \dfrac{e^{i(n+1)\theta} - 1}{e^{i\theta}-1}.\] Nous voulons extraire la partie réelle et imaginaire. Utilisons la factorisation de l’angle moitié : \[U_n = \dfrac{e^{i{\scriptstyle n+1\over\scriptstyle 2}}}{e^{i{\scriptstyle\theta\over\scriptstyle 2}}} \dfrac{\sin\bigl({\scriptstyle n+1\over\scriptstyle 2}\theta\bigr)}{\sin{\scriptstyle\theta\over\scriptstyle 2}} = e^{in{\scriptstyle\theta\over\scriptstyle 2}} \dfrac{\sin\bigl({\scriptstyle n+1\over\scriptstyle 2}\theta\bigr)}{\sin\bigl({\scriptstyle\theta\over\scriptstyle 2} \bigr)}\] d’où \[S_n = \dfrac{\cos{\scriptstyle n\theta\over\scriptstyle 2} \sin{\scriptstyle(n+1)\theta\over\scriptstyle 2}}{\sin{\scriptstyle\theta\over\scriptstyle 2}} \quad T_n = \dfrac{\sin{\scriptstyle n\theta\over\scriptstyle 2} \sin {\scriptstyle(n+1)\theta\over\scriptstyle 2}}{\sin{\scriptstyle\theta\over\scriptstyle 2}}.\]

  2. Si \(e^{i\theta} = 1\), c’est-à-dire \(\theta = 2k\pi\)\(k\in\mathbb{Z}\), alors \(U_n = n+1\) d’où \(S_n = (n+1)\) et \(T_n = 0\).

Vous devez penser par vous-même à introduire la somme complexe correspondante. Par exemple, pour calculer la somme \[S_n = \displaystyle{\sum_{k=0}^{n}} \dbinom{n}{k} \cos(\alpha + k \beta)\]\(\alpha, \beta \in \mathbb{R}\), introduisez la somme des exponentielles imaginaires correspondante : \[U_n = \displaystyle{\sum_{k=0}^{n}} \dbinom{n}{k} e^{i(\alpha + k \beta)}\] puisque \(S_n = \mathop{\mathrm{Re}}(U_n)\). Le calcul de \(U_n\) se ramène au calcul d’une somme binômiale : \[U_n = e^{i\alpha} \displaystyle{\sum_{k=0}^{n}} \dbinom{n}{k} \bigl[e^{i\beta}\bigr]^k = e^{i\alpha} \bigl[e^{i\beta} + 1\bigr]^n.\] Il suffit alors de factoriser l’angle moitié : \[U_n = e^{i\alpha} \Bigl[2\cos{\scriptstyle\beta\over\scriptstyle 2}e^{i{\scriptstyle\beta\over\scriptstyle 2}}\Bigr]^n = 2^n \cos^n {\scriptstyle\beta\over\scriptstyle 2} e^{i(\alpha + {\scriptstyle n\beta\over\scriptstyle 2})}\] et de prendre la partie réelle : \[S_n = 2^n \cos\bigl(\alpha + \dfrac{n\beta}{2}\bigr)\cos^n \dfrac{\beta}{2}.\]

Un dernier exemple : nous voulons calculer la somme \[S_n = \displaystyle{\sum_{k=0}^{n}} \cos^2(k\theta)\] Nous ne pouvons pas ici introduire de somme d’exponentielles imaginaires associés. Si par exemple \(U_n = \displaystyle{\sum_{k=0}^{n}} \bigl[e^{ik\theta}]^2\), nous n’avons pas \(S_n = \mathop{\mathrm{Re}}(U_n)\) ! Pour calculer cette somme, il suffit de linéariser \(\cos^2(k\theta) = \dfrac{1}{2}\bigl[\cos(2k\theta) + 1\bigr]\) et on a alors \[S_n = \dfrac{1}{2} \displaystyle{\sum_{k=0}^{n}} \cos(2k\theta) + \dfrac{(n+1)}{2}.\] Vous pouvez terminer le calcul en introduisant la somme complexe \(U_n = \displaystyle{\sum_{k=0}^{n}} e^{2ik\theta}\).

Calculs sur des polynômes

Les trinômes

Le but de ce paragraphe est de rappeler les principales propriétés des trinômes qui doivent être parfaitement connues.

Discriminant réduit

On considère un polynôme du second degré \[P = a X^2 + b X + c \quad \textrm{ avec } a\neq 0.\] On sait calculer ses deux racines complexes \(x_1,x_2\) à l’aide du discriminant : \[\Delta = b^2 - 4ac\]

  • Si \(\Delta = 0\), le trinôme possède une racine double donnée par \(x = -\dfrac{b}{a}\).

  • Si \(\Delta \neq 0\), le trinôme possède deux racines complexes distinctes. On commence par calculer une racine carrée complexe de \(\Delta\), c’est-à-dire un complexe \(\delta\) vérifiant \(\delta^2 = \Delta\) et on obtient \[\begin{cases} x_1 &= \dfrac{-b - \delta}{2a}, \\[0.5cm] x_2 &= \dfrac{-b + \delta}{2a}. \end{cases}\]

  • Si \(\Delta < 0\) est un réel strictement négatif, on sait que les deux racines complexes sont conjuguées : \(x_2 = \overline{x_1}\).

Lorsque le trinôme s’écrit \[P = aX^2 + 2bX + c\] (utile lorsque \(b\) est entier), on peut utiliser directement le discriminant réduit \(\Delta' = b^2 - ac\) et exprimer les racines à l’aide d’une racine carrée complexe de \(\Delta'\) \((\delta'^2 = \Delta'\)) : \[\begin{cases} x_1 &= \dfrac{-b-\delta'}{a}, \newline[0.5cm] x_2 &= \dfrac{-b+\delta'}{a}. \end{cases}\] Cela évite la simplification par \(2\) dans les expressions …

Pour résoudre l’équation \(x^2 -4x + 3 = 0\), on écrit \(\Delta' = 2^2 - 3 = 1\) et les deux racines sont \(x_1 = 2+1 = 3\) et \(x_2 = 2-1 = 1\).
Une démonstration de l’inégalité de Cauchy-Schwarz utilise le discriminant réduit. Soit \(E\) un espace euclidien et \(x,y \in E\) deux vecteurs, considérons (lorsque \(x \neq 0_E\)) le trinôme \[T(\lambda) = \lVert \lambda x + y \rVert_{ }^2 = \lambda^2 \lVert x \rVert_{ }^2 + 2\lambda \left( x \mid y \right) + \lVert y \rVert_{ }^2.\] Puisqu’une norme ne prend que des valeurs positives, ce trinôme ne prend que des valeurs positives et ne possède donc au plus qu’une racine réelle. Son discriminant réduit est négatif ou nul : \[\Delta' = \left( x \mid y \right)^2 - \lVert x \rVert_{ }^2 \lVert y \rVert_{ }^2 \leqslant 0\] d’où \(\lvert \left( x \mid y \right) \rvert \leqslant\lVert x \rVert_{ }\lVert y \rVert_{ }\).

Relations entre coefficients et racines

En notant \(x_1,x_2\) les deux racines complexes du polynôme \(P = aX^2+bX+c\), on a \[P = a(X-x_1)(X-x_2) = a\bigl[ X^2 - (x_1+x_2) X + x_1x_2 \bigr].\] En identifiant les coefficients de \(P\), on déduit la somme et le produit des racines sans avoir à les calculer explicitement : \(\heartsuit\) \[\begin{aligned} \begin{cases} x_1 + x_2 &= -\dfrac{b}{a}, \newline[0.5cm] x_1x_2 &= \dfrac{c}{a}. \end{cases} \end{aligned}\]

Déterminons l’expression logarithmique de \(\mathop{\mathrm{argsh}}(x)\). Soit \(x \in \mathbb{R}\), notons \(y = \mathop{\mathrm{argsh}}(x)\) : \[x = \mathop{\mathrm{sh}}(y) = \dfrac{e^y - e^{-y}}{2}.\] En notant \(Y = e^y\), \(Y\) est racine d’un trinôme : \(x = \dfrac{Y - 1/Y}{2} = \dfrac{Y^2-1}{2Y}\). On en déduit que \[Y^2 - 2x Y - 1 = 0.\] Le discriminant réduit vaut \(\Delta' = x^2+1 > 0\) et le trinôme possède deux racines réelles distinctes \[\begin{cases} Y_1 &= x + \sqrt{x^2+1} > 0, \\ Y_2 &= x - \sqrt{x^2 - 1} < 0. \end{cases}\] En effet, puisque \(Y_1Y_2 = -1 < 0\), l’une des racines est positive, l’autre est négative et puisque de façon évidente \(Y_1 > 0\), il vient que \(e^y = x + \sqrt{x^2+1}\) d’où \[\mathop{\mathrm{argsh}}(x) = \ln(x + \sqrt{x^2+1}).\] Déterminons de même l’expression logarithmique de \(y=\mathop{\mathrm{argch}}(x)\) pour \(x \geqslant 1\). Avec les mêmes notations, \[x = \dfrac{e^y + e^{-y}}{2} = \dfrac{Y+1/Y}{2} = \dfrac{Y^2+1}{2}\] et \[Y^2 - 2xY + 1 = 0.\] Le discriminant réduit \(\Delta' = x^2-1 \geqslant 0\) est toujours positif d’où les deux racines \[\begin{cases} Y_1 &= x + \sqrt{x^2-1}, \geqslant 1 \\ Y_2 &= x - \sqrt{x^2+1}. \end{cases}\] Puisque \(Y_1Y_2 = 1\), les deux racines ont même signe (\(>0\)), l’une est supérieure et l’autre inférieure à \(1\). Puisque \(Y_1 \geqslant x \geqslant 1\), et que \(y = \mathop{\mathrm{argch}}(x) = \ln Y \geqslant 0\), il vient \(Y \geqslant 1\) d’où \(Y = Y_1\) et \(\mathop{\mathrm{argch}}(x) = \ln(x + \sqrt{x^2-1})\).

Extrémum d’un trinôme

La courbe représentative d’un trinôme \(y = P(x) = ax^2 + bx + c\) est une parabole.

  1. \(a > 0\) : la parabole est convexe et atteint son maximum en \(c = -\dfrac{b}{a}\) : le milieu des racines de \(P\) (lorsque \(P\) possède deux racines réelles).

  2. \(a < 0\) : la parabole est concave et atteint son minimum en \(c = -\dfrac{b}{a}\) : le milieu des racines de \(P\) (lorsque \(P\) possède deux racines réelles).

@ >p(- 2) * >p(- 2) * @

(-1,-1.5)(3,1.5) (-1,0)(3,0) (1,0)\(c={\scriptstyle\alpha+\beta\over\scriptstyle 2}\) (0,0)\(\alpha\) (2,0)\(\beta\) (0.5,1)(1.5,1) (1,0)

&

(-1,-1.5)(3,1.5) (-1,0)(3,0) (1,0)\(c={\scriptstyle\alpha+\beta\over\scriptstyle 2}\) (0,0)\(\alpha\) (2,0)\(\beta\) (0.5,-1)(1.5,-1) (1,0)


\(P(x) = (x-\alpha)(\beta-x)\) & \(P(x) = (x-\alpha)(x-\beta)\)

Il suffit d’étudier les variations de \(P\) : \(P'(x) = 2ax + b\) et \(P'\) s’annule en \(-b/a\).

\(\displaystyle{\sup_{x\in[0,1]} x(1-x)} = \dfrac{1}{4}\) Le trinôme admet un maximum atteint en \(x = 1/2\) (milieu des racines \(0\), \(1\)) et la valeur du maximum vaut \(1/4\).
(2).
On considère la suite de terme général \[u_n = \dfrac{1}{n^3} \displaystyle{\sum_{k=1}^{n}} \sqrt{k(n-k)}\]
  1. Montrer que \(u_n \xrightarrow[n \rightarrow +\infty]{}0\).

  2. Déterminer un équivalent simple de \(u_n\).

  1. Le trinôme \(P=X(n-X)\) a pour racines \(0\) et \(n\) et atteint son maximum sur \([0, n]\) en \(c = n/2\). Le maximum vaut \(n^2/4\). On majore facilement : \[0 \leqslant u_n \leqslant\dfrac{1}{n^3} n \times \sqrt{n^2/4} = \dfrac{1}{2n}\] et \(u_n \xrightarrow[n \rightarrow +\infty]{}0\).

  2. Écrivons \[u_n = \dfrac{1}{n} \Bigl[\dfrac{1}{n} \displaystyle{\sum_{k=1}^{n}} \sqrt{\dfrac{k}{n}(1-\dfrac{k}{n})}\Bigr] = \dfrac{S_n}{n}\]\(S_n\) est une somme de Riemann qui converge vers \(L = \displaystyle{\int_{0}^{1}} \sqrt{x(1-x)}\mathrm{ \;d}x\) Mais la courbe d’équation \(y = \sqrt{x(1-x)}\) a pour équation \(y^2 = x - x^2\) et on reconnaît un demi-cercle centré en \((1/2,0)\) de rayon \(1/2\). L’intégrale \(L\) est l’aire d’un demi-disque de rayon \(1/2\) et vaut donc \(L = \pi/8\). Finalement, \(u_n \underset{n \rightarrow +\infty}{\sim} \dfrac{\pi}{8n}\).

Développement de polynômes

Pour développer un produit de polynômes, il est indispensable de mener les calculs en une seule ligne. Par exemple, pour développer \[P(X) = (X^2 + 3X - 2)(X^3+2X^2-3X +1)\] il est très maladroit d’écrire : \[\begin{aligned} P(X) &= X^2(X^3 + 2X^2-3X+1) + 3X(X^3+2X^2-3X+1) -2(X^3+2X^2-3X+1) \\ &= X^5 + 2X^4 - 3X^3 + X^2 + 3X^4 + 6X^3 - 9X^2 + 3X - 2X^3 -4 X^2 +6X -2 \newline &= X^5 +5X^4+X^3 -12X^2 + 9X -2. \end{aligned}\] En effet, recopier mécaniquement des termes mène à une perte d’attention, source fréquente d’erreurs.

La bonne méthode pour mener ce calcul consiste à :

  • déterminer le degré \(d\) du polynôme produit (somme des degrés),

  • déterminer pour \(k\in [\kern-0.127em[ 0, d ]\kern-0.127em]\) d’où vient le terme en \(X^k\) et sommer les coefficients sélectionnés,

  • écrire en une seule ligne le résultat.

Multimédia: Coefficients entiers de 2 (ou 3) polynômes choisis au hasard, pour calculer le coefficient de \(X^k\), griser dynamiquement les termes du produit et collecter les coefficients en bas. Exerciseur ?

Détaillons cette technique sur notre exemple. Les opérations suivantes se font de tête et ne nécessitent qu’une seule ligne ! Le degré du polynôme \(P\) vaut \(5\).

  • Le terme en \(X^5\) ne provient que du produit de \(X^2\) avec \(X^3\) et le coefficient de \(X^5\) vaut donc \(1\) : \[(\boxed{X^2} + 3X - 2)(\boxed{X^3}+2X^2-3X +1).\]

  • Le terme en \(X^4\) provient du produit de \(X^2\) avec \(2X^2\) (coefficient \(2\)) \[(\boxed{X^2} + 3X - 2)(X^3+ \boxed{2X}^2-3X +1),\] du produit de \(3X\) avec \(X^3\) (coefficient \(3\)) \[(X^2 + \boxed{3X} - 2)(\boxed{X^3}+2X^2-3X +1)\] et le coefficient de \(X^4\) vaut donc \(2+3 = 5\).

  • Le terme en \(X^3\) provient du produit de \(X^2\) avec \(-3X\), \[(\boxed{X^2} + 3X - 2)(X^3+2X^2 \boxed{-3X} +1),\] du produit de \(3X\) avec \(2X^2\) \[(X^2 + \boxed{3X} - 2)(X^3+ \boxed{2X^2}-3X +1)\] et du produit de \(-2\) avec \(X^3\) \[(X^2 + 3X \boxed{- 2})(\boxed{X^3}+2X^2-3X +1).\] Le coefficient vaut donc \(-3+6-2=1\).

  • Le terme en \(X^2\) a pour coefficient \(1-9-4=-12\) : \[(\boxed{X^2} + 3X - 2)(X^3+2X^2-3X \boxed{+1}),\] \[(X^2 + \boxed{3X} - 2)(X^3+2X^2 \boxed{-3X} +1),\] \[(X^2 + 3X \boxed{- 2})(X^3+\boxed{2X^2}-3X +1).\]

  • Le terme en \(X\) a pour coefficient \(3+6=9\) \[(X^2 + \boxed{3X} - 2)(X^3+2X^2-3X \boxed{+1}),\] \[(X^2 + 3X \boxed{- 2})(X^3+2X^2\boxed{-3X} +1).\]

  • Le terme constant provient du produit des deux constantes et vaut \(-2\). \[(X^2 + 3X \boxed{- 2})(X^3+2X^2-3X \boxed{+1}).\]

Il faut impérativement adopter cette technique de calcul. Dans un premier temps, vous pouvez utiliser le brouillon pour noter les coefficients que vous récoltez et ensuite les sommer. Avec un peu d’habitude, ce calcul se fait de tête immédiatement.

Développer les polynômes suivants :
  1. \((X^3+X^2-X+2)(2X^2+X-1)\);

  2. \((2X^4+3X^2-6X-1)(X^3-2X^2+X+1)\);

  3. \((X^5+X^4-2X^2+3)(2X^3-X^2+X-1)\).

  1. \(2X^5+3X^4-2X^2+3X-2\);

  2. \(2X^7-4X^6+5X^5-10X^4+14X^3-X^2-7X-1\);

  3. \(2X^8+X^7-4X^5+X^4+4X^3-X^2+3X-3\).

Déterminer le développement limité à l’ordre \(4\) en \(0\) de :
  1. \(\sin(x) \exp(x)\);

  2. \(\dfrac{\ln(1+x)}{1+x}\).

  1. \[\begin{aligned} e^x \sin(x) &= \bigl(x - {\scriptstyle x^3\over\scriptstyle 6} + \underset{x^4}{\mathrm{o}}(\bigr))\bigl(1+x+{\scriptstyle x^2\over\scriptstyle 2} + {\scriptstyle x^3\over\scriptstyle 6} + \underset{x^3}{\mathrm{o}}(\bigr)) \\ &= x\bigl(1-{\scriptstyle x^2\over\scriptstyle 6}+\underset{x^3}{\mathrm{o}}())(1+x+{\scriptstyle x^2\over\scriptstyle 2}+{\scriptstyle x^3\over\scriptstyle 6} + \underset{x^3}{\mathrm{o}}()) \\ &= x + x^2 + {\scriptstyle x^3\over\scriptstyle 3} + \underset{x^3}{\mathrm{o}}(.) \end{aligned}\]

  2. \[\begin{aligned} \dfrac{\ln(1+x)}{1+x} &= x(1-{\scriptstyle x\over\scriptstyle 2} + {\scriptstyle x^2\over\scriptstyle 3} - {\scriptstyle x^3\over\scriptstyle 4} + \underset{x^3}{\mathrm{o}}())(1-x+x^2-x^3+\underset{x^3}{\mathrm{o}}()) \\ &= x(1-{\scriptstyle 3\over\scriptstyle 2}x + {\scriptstyle 11\over\scriptstyle 6}x^2 - {\scriptstyle 25\over\scriptstyle 12}x^3 + \underset{x^3}{\mathrm{o}}()) \newline &= x - {\scriptstyle 3\over\scriptstyle 2} x^2 + {\scriptstyle 11\over\scriptstyle 6}x^3 - {\scriptstyle 25\over\scriptstyle 12}x^4 + \underset{x^4}{\mathrm{o}}(.) \end{aligned}\]

Pour développer un produit de trois polynômes, on peut utiliser la même technique et chercher directement d’où provient le terme \(X^d\).

Développer \[P(X) = (X^2+X-1)(X^2+3X-1)(X-5).\] Le degré du produit vaut \(5\).
  • Le terme en \(X^5\) provient uniquement du produit \[(\boxed{X^2}+X-1)(\boxed{X^2}+3X-1)(\boxed{X}-5)\] et vaut \(X^5\).

  • Le terme en \(X^4\) provient des produits : \[(\boxed{X^2}+X-1)(\boxed{X^2}+3X-1)(X \boxed{-5}),\] \[(\boxed{X^2}+X-1)(X^2 + \boxed{3X}-1)(\boxed{X}-5),\] \[(X^2+\boxed{X}-1)(\boxed{X^2}+3X-1)(\boxed{X} -5)\] et le coefficient de \(X^4\) vaut donc \(-5+3+1 = -1\).

  • Terminer le calcul de cette façon.

Si vous n’êtes pas à l’aise avec cette méthode, vous pouvez effectuer le calcul en deux lignes en ne faisant que des multiplications de deux polynômes en utilisant la méthode précédente (une multiplication par ligne ) : \[\begin{aligned} P &= P(X) = (X^2+X-1)(X^2+3X-1)(X-5)\\ &= (X^2+X-1)(X^3-2X^2-16X+5) \newline &= X^5 - X^4 - 19X^3 - 9X^2 + 21X - 5. \end{aligned}\]

Factorisation de polynômes

Il est tout aussi important de savoir factoriser un polynôme. Rappelons que Maple permet de développer et de factoriser les polynômes :

> P := (x+1)*(x^2-3*x+2);
                                    2
                     P := (x + 1) (x  - 3 x + 2)
> expand(P);
                           3      2
                          x  - 2 x  - x + 2
> Q := X^4 + 2*X^3 - 11*X^2 - 12*X + 36;
                        4      3       2
                  Q := X  + 2 X  - 11 X  - 12 X + 36
> factor(Q);
                                 2        2
                          (X + 3)  (X - 2)

La factorisation d’un polynôme revient à déterminer ses racines (complexes). On se heurte ici à l’un des principaux problèmes des mathématiques. Depuis l’antiquité, on connaît un algorithme pour trouver les racines d’un polynôme de degré \(2\) et on sait également déterminer les racines d’un polynôme de degré \(3\) et de degré \(4\).

Manu : note historique + précise avec photo ?

On a cherché longtemps un algorithme similaire permettant d’exprimer les racines d’un polynôme de degré \(5\) à l’aide de radicaux faisant intervenir les coefficients du polynôme. Abel puis Galois ont montré que ce problème était insoluble.

Cette obstruction au calcul de racines d’un polynôme se retrouve dans toutes les branches des mathématiques :

On sait en théorie primitiver une fraction rationnelle. En pratique, il faut être capable de la décomposer en éléments simples. Pour cela, la première étape consiste à factoriser le polynôme au dénominateur. Dès que le degré du dénominateur est supérieur à \(5\) on est bloqué. Voici une réponse de Maple pour le calcul de \(\int \dfrac{\mathrm{ \;d}x}{x^6+x^5+1}\) :
int( 1/(x^6+x^5+1), x);
   -----
    \               336657046555205   5   43328897320370   4
     )    _R ln(x - --------------- _R  - -------------- _R
    /                363446734701          363446734701
   -----
  _R = %1
           7892189829980   3   408178287515   2   2020667424601
         - ------------- _R  - ------------ _R  + ------------- _R
           363446734701        121148911567       363446734701

           43796810471
         + ------------)
           363446734701
                       6         4        3        2
                       % 1 := RootOf(43531 _Z + 540 _Z + 80 _Z + 10 _Z
                       % + _Z + 1)
Ce n’est pas tout à fait ce que l’on appelle une primitive explicite …

On est souvent confronté à cette obstruction fondamentale en calcul scientifique et le seul choix possible consiste à utiliser des méthodes numériques pour déterminer des valeurs approchées des racines d’un polynôme. Heureusement, on dispose d’algorithmes très efficaces pour trouver ces valeurs approchées (vous connaissez déjà la méthode de Newton par exemple ...). Voici un calcul avec Maple qui illustre l’impossibilité de trouver les valeurs exactes des racines d’un polynôme. Une méthode numérique permet cependant de trouver des racines réelles approchées :

P := x^10 -2*x - 1;
                                10
                          P := x   - 2 x - 1
> factor(P);
                             10
                            x   - 2 x - 1
> solve(P = 0, x);
                                10
                       RootOf(_Z   - 2 _Z - 1)
> fsolve(P=0, x);
                      -.4995164207, 1.125099677

Dans la suite de ce paragraphe, nous allons voir quelques techniques qui permettent de factoriser de façon explicite un polynôme dans certains cas très particuliers.

Factoriser à partir d’une racine connue

On veut factoriser un polynôme \(P\) de degré \(d\). Si l’on connaît une racine \(\alpha\) de ce polynôme, on se ramène à la factorisation d’un polynôme \(Q\) de degré \((d-1)\) en factorisant \((X-\alpha)\) dans \(P\). Considérons le polynôme \[P(X) = X^3 - 5X^2 + 7X -3.\] On voit que \(\alpha = 1\) est racine évidente de \(P\) puisque \(1 - 5 \times 1 + 7\times 1 - 3 = 0\). On peut donc écrire \[P = (X-1)Q(X)\] Il faut savoir écrire en une seule ligne le polynôme \(Q\) en utilisant la méthode suivante :

  • \(P = X^3 - 5X^2 + 7X - 3\)

  • Le degré du polynôme \(Q\) vaut \(2\) : \(P = (X-1)(aX^2 + bX + c)\).

  • On détermine le coefficient \(a\) puisque le produit de \(X\) avec \(aX^2\) doit donner le terme en \(X^3\) de \(P\) : \(a = 1\) \[P = (X-1)(X^2 + ?X + \dots).\]

  • On cherche le coefficient de \(X\) dans \(Q\) en examinant le terme \(X^2\) du produit : \((-1+?)X^2 = -5X^2\) d’où \(? = -4\) : \[P = (X-1)(X^2-4X + ?).\]

  • On s’intéresse ensuite au terme en \(X\) dans le développement : \((?+4)X = 7\) d’où \(? = 3\). \[P = (X-1)(X^2-4X+3).\]

  • On vérifie notre calcul en calculant le terme constant du produit : \((-1) \times 3 = -3\) qui est bien le coefficient constant de \(P\).

Voici un autre exemple. Nous détaillons ici les calculs en plusieurs étapes afin d’illustrer la méthode, mais vous devez mener ce calcul de tête en une seule ligne ! \[P = 2X^5 - X^4 -3X^3 + X^2 + 3X - 2.\] On voit immédiatement que \(1\) est racine évidente de \(P\) (la somme des coefficients est nulle) et on factorise en une seule ligne \(P\) en suivant les étapes suivantes :

  • \(P = (X-1)(2X^4 \dots)\),

  • \(P = (X-1)(2X^4 + X^3 \dots)\),

  • \(P = (X-1)(2X^4 + X^3 - 2X^2 \dots)\),

  • \(P = (X-1)(2X^4 + X^3 - 2X^2 -X \dots)\),

  • \(P = (X-1)(2X^4 + X^3 - 2X^2 - X + 2)\),

  • On vérifie le coefficient constant.

Multimédia: Applet exerciseur

Trouver toutes les racines rationnelles

La méthode précédente permet de factoriser \((X-\alpha)\)\(\alpha\) est une racine de \(P\). Encore faut-il être capable de trouver une racine évidente   du polynôme \(P\) ! Pour un polynôme quelconque, c’est impossible comme nous l’avons vu dans l’introduction. Il existe tout de même quelques méthodes à connaître.

D’abord des remarques évidentes :

  • Si le coefficient constant est nul, on peut factoriser \(X\) ! \[X^5 + 4X^2 - 5X = X(X^4 + 4X - 5) = X(X-1)(X^3+X^2+X+5)\] \[X^6 - 4X^5 + 3X^3 = X^3(X^3 - 4X^2 + 3) = X^3(X-1)(X^2-3X-3) = X^3(X-1)(X-\alpha)(X-\beta)\]\(\alpha = {\scriptstyle 3+\sqrt{21}\over\scriptstyle 2}\) et \(\beta={\scriptstyle 3-\sqrt{21}\over\scriptstyle 2}\).

  • Examiner les coefficients du polynôme : si leur somme vaut \(0\) alors \(1\) est racine évidente. Regarder de même si \(P(-1) = 0\).

Il existe un algorithme pour déterminer les racines rationnelles d’un polynôme à coefficients rationnels. Voir exercice [racines_rationnelles] p. [racines_rationnelles] Considérons par exemple le polynôme \[P = X^3 - \dfrac{7}{6}X^2 - \dfrac{3}{2}X + \dfrac{3}{2}.\]

  1. On peut supposer que le coefficient constant \(a_0\) est non-nul, sinon on peut factoriser \(X^k\) dans \(P\) et se ramener à chercher les racines d’un polynôme de degré inférieur.

  2. On peut se ramener à chercher les racines rationnelles d’un polynôme à coefficients entiers en réduisant les fractions au même dénominateur : \[P = \dfrac{a_d}{b_d} X^d + \dots + \dfrac{a_1}{b_1} X + \dfrac{a_0}{b_0} = \dfrac{\alpha}{\beta}( b_{d-1} \dots b_0 X^d + \dots + b_d\dots b_1).\] Sur notre exemple, \[P = \dfrac{1}{6}(6X^3 - 7X^2 -9X + 9).\] On se ramène ainsi à chercher les racines du polynôme \[Q = 6X^3 -7X^2 -9X + 9.\]

  3. Supposons que \(\alpha = p/q\) soit une racine rationnelle du polynôme \(Q = a_d X^d + a_{d-1} X^{d-1} + \dots + a_1X + a_0\)\(a_i \in \mathbb{Z}\). On suppose que \(p \in \mathbb{Z}\) et \(q \in \mathbb{N}^*\) et que les entiers \(p\) et \(q\) sont premiers entre eux : \(p\wedge q = 1\). On doit avoir : \[0 = Q(\alpha) = a_d \dfrac{p^d}{q^d} + a_{d-1}\dfrac{p^{d-1}}{q^{d-1}} + \dots + a_1 \dfrac{p}{q} + a_0\] et, en réduisant au même dénominateur, \[a_d p^d + a_{d-1} qp^{d-1} + \dots + a_1 q^{d-1}p + a_0q^d = 0.\] On obtient ainsi une relation entre entiers et on peut utiliser le cours d’arithmétique dans \(\mathbb{Z}\).

  4. Écrivons en factorisant \(p\) : \[p( -a_d p^{d-1} - \dots - a_1 q^{d-1}) = a_0 q^d.\] On en déduit que l’entier \(p\) doit diviser l’entier \(a_0 q^d\) et puisque \(p \wedge q = 1\), on a aussi \(p \wedge q^d = 1\) d’où en utilisant le théorème de Gauss : \[\begin{cases} p \mid a_0 q^d \\ p \wedge q^d = 1 \end{cases} \underbrace{\Rightarrow }_{\textrm{ Gauss}} p \mid a_0.\] L’entier \(p\) doit diviser le coefficient \(a_0\). L’entier \(p\) ne peut donc prendre qu’un nombre fini de valeurs, les diviseurs de \(a_0\). Sur notre exemple, \(p \in \{1,3,9,-1,-3,-9\}\).

  5. De façon symétrique, puisque \[a_d p^d = q(-a_{d-1}p^{d-1} - \dots - a_0 q^{d-1}),\] on a \(q \mid a_d p^d\) et, puisque \(q\wedge p = 1\), il vient \(q \mid a_d\) et \(q\) ne peut prendre qu’un nombre fini de valeurs. Sur notre exemple, \(q \in \{1,2,3,6\}\)

  6. On ne peut donc avoir qu’un nombre fini de racines rationnelles de \(P\), sur notre exemple \[\alpha \in \left\{1,3,9,\dfrac{1}{2},\dfrac{3}{2},\dfrac{9}{2},\dfrac{1}{3},\dfrac{1}{6}, -1,-3,-9,-\dfrac{1}{2},-\dfrac{3}{2},-\dfrac{9}{2},-\dfrac{1}{3},-\dfrac{1}{6}\right\} .\]

  7. Il suffit, pour chacun de ces rationnels, de tester si \(P(\alpha)=0\). On en déduit toutes les racines rationnelles de \(P\). Sur notre exemple, \(\alpha = 3/2\) est la seule racine rationnelle de \(P\).

Polynômes particuliers

Polynômes bicarrés

Si un polynôme s’écrit \(P = Q(X^k)\), il suffit, pour trouver les racines de \(P\), de déterminer les racines de \(Q\). En effet, si \(P(\alpha) = 0\), alors \(\alpha^k\) est une racine de \(Q\) et on sait déterminer les racines \(k\)-ièmes d’un nombre complexe. Par exemple, \[P = X^6 -3X^3 + 2 = (X^3)^2 - 3(X^3) + 2 = Q(X^3) \textrm{ où } Q(X) = X^2 - 3X + 2 = (X-1)(X-2)\] Les racines de \(P\) sont donc \(\left\{1,j, j^2, \sqrt[3]{2}, j\sqrt[3]{2}, j^2\sqrt[3]{2}\right\}\).

Un polynôme bicarré est un polynôme \(P = Q(X^2)\)\(Q\) est un polynôme du second degré : \(Q(X) = aX^2+bX+c\). Par exemple, \[P(X) = X^4 - 3X^2 + 2 = Q(X^2) \textrm{ où } Q(X) = X^2-3X+2 = (X-1)(X-2)\]

Dans le chapitre [chapitre_polynomes], on a vu que les polynômes irréductibles normalisés dans \(\mathbb{\mathbb{R} }_{ }[X]\) étaient :

  • les polynômes du premier degré \((X-\alpha)\),

  • les polynômes du second degré sans racines réelles : \(X^2 + pX + q\) avec \(\Delta = p^2 - 4q < 0\).

Un polynôme bicarré n’est donc jamais irréductible et on peut toujours le factoriser ! On peut évidemment déterminer les racines complexes \(\alpha, \beta\) de \(Q\) et les \(4\) racines complexes de \(P\) seront les racines carrées complexes de \(\alpha\) et \(\beta\). Pour obtenir la décomposition dans \(\mathbb{\mathbb{R} }_{ }[X]\), il suffit alors de grouper ces racines conjuguées deux à deux. Cette méthode est souvent lourde et il faut connaître l’algorithme suivant qui permet d’obtenir directement la factorisation de \(P\) dans \(\mathbb{\mathbb{R} }_{ }[X]\) sans passer par les complexes.

  1. Si le polynôme \(Q(X) = (X-a)(X-b)\) possède deux racines réelles, \(P(X) = (X^2-a)(X^2-b)\), selon le signe de \(a\) et \(b\), on obtient directement la factorisation de \(P\) dans \(\mathbb{\mathbb{R} }_{ }[X]\). Par exemple, \[P(X) = (X^2-1)(X^2-2) = (X-1)(X+1)(X-\sqrt{2})(X+\sqrt{2}),\] \[P(X) = (X^2+3)(X^2-1) = (X^2+3)(X-1)(X+1).\]

  2. Si le polynôme \(Q\) ne possède pas de racines réelles, travailler directement sur le polynôme \(P\) en groupant le terme en \(X^4\) avec le terme constant et en faisant apparaître un début de carré et en utilisant la factorisation \(a^2-b^2 = (a-b)(a+b)\) :

    \[P = X^4 + X^2 + 1 = (X^4 + 1) + X^2 = (X^2+1)^2 -X^2 = (X^2-X+1)(X^2+X+1),\] \[P = X^4 + 1 = (X^2+1)^2 - 2X^2 = (X^2 - \sqrt{2}X + 1)(X^2+\sqrt{2}X + 1),\] \[\begin{aligned} P &= X^4 + 2X^2 + 2 = (X^4+2) + 2X^2 = (X^2+\sqrt{2})^2 + (2-2\sqrt{2})X^2 \newline & = (X^2 - \sqrt{2-2\sqrt{2}}X + \sqrt{2})(X^2 + \sqrt{2-2\sqrt{2}}X + \sqrt{2}). \end{aligned}\]

Racines de l’unité

Certains polynômes font intervenir les racines \(n\)-ièmes de l’unité et se factorisent facilement.

Le polynôme \(P = 1 + X + X^2\) a pour racines \(j\) et \(j^2\) : \(P = (X-j)(X-j^2)\). Il faut le reconnaître et le factoriser sans calculer son discriminant !
Le polynôme \(Q = X^2 - X + 1\) a pour racines \(-j\) et \(-j^2\) puisque \(Q(-X) = P(X)\).

Plus généralement, considérons le polynôme \(P = X^n - 1\). Ses racines complexes sont les \(n\) racines \(n\)-ièmes de l’unité : \(\omega_k = e^{{\scriptstyle 2ik\pi\over\scriptstyle n}}\), \(k \in [\kern-0.127em[ 0, n-1 ]\kern-0.127em]\). On a l’identité remarquable \(X^n - 1 = (X-1)(1+X+X^2+\dots + X^{n-1})\). Il en découle que le polynôme \[Q = 1 + X + \dots + X^{n-1}\] a pour racines les racines \(n\)-ièmes de l’unité sauf \(1\).

On sait également écrire la factorisation dans \(\mathbb{\mathbb{R} }_{ }[X]\) du polynôme \(P = X^n - 1\) en groupant les racines \(n\)-ièmes de l’unité avec leurs conjugués. On a toujours \(P(1) = 0\) et selon la parité de \(n\) :

  • si \(n = 2p\) est pair, \(P(-1) = 0\) et \(-1\) est une racine \(n\)-ième de l’unité. Les \((2p-2)\) autres racines ne sont pas réelles et puisque le conjugué de \(\omega_k\) est \(\omega_{2p-k}\), on peut écrire \[X^{2p} - 1 = (X-1)(X+1) \displaystyle{\prod_{k=1}^{p-1}} \left(X-\omega_k\right)\left(X-\overline{\omega_k}\right) = \left(X-1\right)\left(X+1\right) \displaystyle{\prod_{k=1}^{p-1}} \left(X^2 - 2\cos\left({\scriptstyle 2k\pi\over\scriptstyle n}\right) X + 1\right).\]

  • si \(n = 2p+1\) est impair, \(-1\) n’est pas une racine de l’unité et en groupant les \(2p\) racines différentes de \(1\), on trouve \[X^{2p+1} - 1 = (X-1) \displaystyle{\prod_{k=1}^{p}} \left(X^2 - 2\cos\left(\dfrac{k\pi}{n}\right) X + 1\right).\]

Polynômes réciproques

Cette technique est beaucoup moins importante que les précédentes. Vous pouvez la sauter en première lecture.

Un polynôme \(P = \displaystyle{\sum_{i=0}^{d}} a_i X^i\) est dit réciproque lorsque ses coefficients sont symétriques : \(\forall i \in [\kern-0.127em[ 0, d ]\kern-0.127em]\), \(a_i = a_{d-i}\). Par exemple, le polynôme \[P = 6X^4 + 5X^3 - 38X^2 + 5X + 6\] est réciproque (\(a_4 = a_0\), \(a_3 = a_1\), \(a_2=a_2\)).

  • Une remarque importante : un polynôme est réciproque si et seulement si \[\forall x \neq 0, \quad P\left(\dfrac{1}{x}\right) = \dfrac{P(x)}{x^d}.\] Sur notre exemple, \[P\left(\dfrac{1}{x}\right) = \dfrac{6}{x^4} + \dfrac{5}{x^3} - \dfrac{38}{x^2} + \dfrac{5}{x} + 6 = \dfrac{6+5x^3-38x^2+5x+6}{x^4} = \dfrac{P(x)}{x^4}.\] On en déduit que si \(x\neq 0\) est une racine de \(P\), alors \(1/x\) est également une racine de \(P\). Il suffit donc de trouver deux racines de \(P\) pour les connaître toutes.

  • Voyons une technique pour calculer les racines de \(P\) qui réduit le problème à chercher les racines d’un polynôme de degré \(d/2\) lorsque \(d\) est pair. Nous allons illustrer la technique sur notre exemple. Une fois l’idée comprise, elle se généralise sans problème à tous les polynômes à coefficients symétriques. Soit \(x\neq 0\) une racine de \(P\), en divisant par \(x^2\), on obtient \[0 = 6x^2 + 5x - 38 + \dfrac{5}{x} + \dfrac{6}{x^2} = 6\left(x^2+\dfrac{1}{x^2}\right) + 5\left(x + \dfrac{1}{x}\right) - 38.\] Posons \(y = x + \dfrac{1}{x}\), calculons \[y^2 = x^2 + 2 + \dfrac{1}{x^2} \textrm{ d'où } x^2 + \dfrac{1}{x^2} = y^2 - 2\] et \(y\) doit donc être racine d’un polynôme de degré \(2\) : \[0 = 6(y^2-2) + 5y - 38 = 6y^2 +5y - 50 = 0.\] On cherche les racines de ce trinôme. On trouve \(y = 5/2\) ou \(y = -10/3\) et il suffit ensuite de résoudre deux équations du second degré en \(x\) : \[\begin{cases} x+\dfrac{1}{x} = \dfrac{5}{2},\\[0.5cm] x+\dfrac{1}{x} = -\dfrac{10}{2}, \end{cases} \textrm{ d'où } \begin{cases} x^2 -\dfrac{5}{2}x + 1 = 0, \newline[0.5cm] x^2 + \dfrac{10}{3}x + 1 = 0. \end{cases}\]

    On trouve finalement \(x \in \{ 2, 1/2, -3, -1/3 \}\), d’où la factorisation de notre polynôme, \(P = (X-2)(X-1/2)(X+3)(X+3)(X+1/3)\).

Lorsque le degré \(d\) est impair, il est facile de voir que \(P(-1) = 0\) et, en factorisant \((X+1)\) dans \(P\), on se ramène à un polynôme de degré \((d-1)\) qui est encore réciproque. En effet, \(P(X) = (X+1)Q(X)\), \[\dfrac{(X+1)Q(X)}{X^d} = \dfrac{P(X)}{X^d} = P(1/X) = (1/X + 1) Q(1/X) = \dfrac{X+1}{X}Q(1/X),\] d’où l’on tire \(Q(1/X) = \dfrac{Q(X)}{X^{d-1}}\) et on en déduit que \(Q\) est réciproque.

Factoriser le polynôme \(P = X^5 + 3X^4 + X^3 + X^2 + 3X + 1\).
On factorise \((X+1)\), \(P = (X+1)(X^4+2X^3-X^2+2X+1)\) et on se ramène à factoriser le polynôme réciproque \(Q = X^4+2X^3-X^2+2X+1\). Soit \(x \in \mathbb{C}\) une racine de \(Q\), en divisant par \(x^2\), on obtient \[\left(x^2+\dfrac{1}{x^2}\right) + 2\left(x + \dfrac{1}{x}\right) - 1 = 0\] et en posant \(y = x+1/x\), \(y\) vérifie l’équation du second degré \(y^2 + 2y - 3 = 0\), d’où \(y = 1\) ou \(y = -3\). On résout alors \(x+1/x = 1\), soit \(x^2-x+1 = 0\) d’où \(x_1 = -j\) et \(x_2 = -j^2\), puis \(x+1/x = -3\) et on trouve \(x_3 = (-3+\sqrt{5})/2\) et \(x_4 = (-3-\sqrt{5})/2\) (on vérifie à l’aide des quantités conjuguées que \(1/x_3 = x_4\)). Finalement, \[P = (X+1)(X+j)(X+j^2)\left(X+{\scriptstyle 3-\sqrt{5}\over\scriptstyle 2}\right)\left(X+{\scriptstyle 3+\sqrt{5}\over\scriptstyle 2}\right).\]

Relations entre coefficients et racines

Le but de ce paragraphe est de présenter de façon plus élémentaire, avec des exemples, les relations entre coefficients et racines d’un polynôme et de montrer des applications en calcul algébrique.

Commençons par l’exemple élémentaire d’un trinôme ayant pour racines (complexes) \(\alpha\) et \(\beta\). Il peut s’écrire sous forme développée et sous forme factorisée : \[P = a_2 X^2 + a_1 X + a_0,\] \[P = a_2(X-\alpha)(X-\beta) = a_2\bigl[X^2 - (\alpha+\beta)X + \alpha\beta)\bigr].\] En identifiant les coefficients, on trouve les relations coefficients racines bien connues : \[\begin{cases} \sigma_1 = \alpha + \beta &= -\dfrac{a_1}{a_2} \\[0.5cm] \sigma_2 = \alpha\beta &= \dfrac{a_0}{a_2} \end{cases}.\]

Pour un polynôme de degré \(3\), ayant trois racines complexes (pas nécessairement distinctes) \(\alpha,\beta, \gamma\), le même calcul donne : \[P = a_3X^3 + a_2 X^2 + a_1 X + a_0\] et \[\begin{aligned} P &= a_3(X-\alpha)(X-\beta)(X-\gamma) \\ &= a_3\bigl[ X^3 - (\alpha+\beta+\gamma)X^2 + (\alpha\beta + \alpha\gamma + \beta\gamma)X - \alpha\beta\gamma\bigr],\end{aligned}\] d’où les relations entre coefficients et racines : \[\begin{cases} \sigma_1 = \alpha+\beta+\gamma &= -\dfrac{a_2}{a_3} \\[0.5cm] \sigma_2 = \alpha\beta + \alpha\gamma + \beta\gamma &= \dfrac{a_1}{a_3} \\[0.5cm] \sigma_3 = \alpha\beta\gamma &= -\dfrac{a_0}{a_3} \end{cases}.\]

Un polynôme de degré \(n\) \[P = a_n X^n + a_{n-1} X^{n-1} + \dots + a_1 X + a_0\] s’écrit sous forme factorisée dans \(\mathbb{C}\) \[P = a_n(X-x_1)(X-x_2)\dots(X-x_n)\]\(x_1,\dots, x_n\) sont ses racines (pas nécessairement distinctes). En développant cette forme factorisée et en identifiant les coefficients, on trouve les relations coefficients racines générales : \[\begin{cases} \sigma_1 = x_1+\dots + x_n &= -\dfrac{a_{n-1}}{a_n} \\[0.5cm] \sigma_2 = x_1x_2 + \dots + x_1x_n + x_2x_3 + \dots + x_2x_n + \dots + x_{n-1}x_n &= \dfrac{a_{n-2}}{a_n} \\[0.5cm] \quad~\vdots & \vdots\\ \sigma_n = x_1\dots x_n &= (-1)^n \dfrac{a_0}{a_n}. \end{cases}\]

Ces relations sont importantes car on a vu qu’on ne savait pas exprimer en général les racines d’un polynôme de degré \(\geqslant 5\) à l’aide des coefficients. Par contre, certaines expressions faisant intervenir les racines du polynôme peuvent s’exprimer à l’aide des coefficients. Par exemple, la somme ou le produit des racines et plus généralement tous les \(\sigma_i\) s’expriment à l’aide des coefficients. Il est bon de connaître le résultat suivant (hors programme).

Si l’on prend l’expression des \(\sigma_i\), par exemple lorsque \(n = 3\), \(\sigma_2 = x_1x_2 + x_1x_3 + x_2x_3\), on s’aperçoit que cette expression est invariante par permutation des racines (on peut par exemple échanger les indices \(1\) et \(2\), on retrouve la même expression). L’expression \[N_2 = x_1^2 + x_2^2 + x_3^2\] et, plus généralement, les sommes de Newton \[N_k = x_1^k + x_2^k + x_3^k\] sont invariantes par permutation des racines. L’expression \[E = x_1x_3 + x_2x_3 + x_1^2\] par contre n’est pas invariante.

Il existe un théorème d’algèbre qui dit que toute expression polynomiale en \((x_1,\dots, x_n)\) (c’est-à-dire une somme-produit des \(x_i\)) invariante par permutation des racines peut s’exprimer comme un polynôme en \(\sigma_1,\dots, \sigma_n\).

Prenons par exemple la somme de Newton \(N_2\) et exprimons-la explicitement à l’aide de \(\sigma_1,\sigma_2,\sigma_3\). Calculons \[\sigma_1^2 = (x_1+x_2+x_3)^2 = x_1^2+x_2^2+x_3^2 - 2(x_1x_2+x_1x_3+x_2x_3).\] On trouve donc \(N_2 = \sigma_1^2 - 2\sigma_2\). Calculons ensuite \[\begin{aligned} \sigma_1N_2 &= (x_1+x_2+x_3)(x_1^2+x_2^2+x_3^2) \\ &= N_3 - x_1x_2(x_1+x_2) - x_1x_3(x_1+x_3)-x_2x_3(x_2+x_3) \\ &= N_3 - x_1x_2(x_1+x_2+x_3) - x_1x_3(x_1+x_2+x_3) - x_2x_3(x_1+x_2+x_3) + 3x_1x_2x_3 \newline &= N_3 - \sigma_1\sigma_2 + 3\sigma_3,\end{aligned}\] d’où l’on tire \[N_3 = \sigma_1(\sigma_1^2 - 2\sigma_2) + \sigma_1\sigma_2 - 3\sigma_3 = \sigma_1^3 - 3\sigma_1\sigma_2 + 3\sigma_3.\]

(2).
Soit \(P(X)= X^3+X-1 \in \mathbb{C}_{ }[X]\). On note \(x_1,x_2,x_3\) ses trois racines complexes.
  1. Vérifier (sans chercher à les calculer) que les trois racines sont distinctes.

  2. Effectuer la division euclidienne de \(X^5\) par \(P\).

  3. En déduire la valeur de la somme \[N_5 = \sum_{k=1}^3 x_k^5 .\]

  1. Par l’absurde, si \(x\) est une racine double de \(P\), alors \(P(x)=P'(x)=0\). Mais \(P'(x)=0 \Rightarrow 3x^2+1=0 \Rightarrow x=\pm \dfrac{i}{\sqrt{3}}\), qui ne sont pas des racines de \(P\). Donc toutes les racines complexes de \(P\) sont simples.

  2. On trouve \(X^5=(X^2-1)P(X)+X^2+X-1\).

  3. Si \(x_k\) est une racine de \(P\), alors \(x_k^5=(x_k^2-1)P(x_k)+x_k^2+x_k-1 = x_k^2+x_k-1\). On peut alors exprimer la somme cherchée en fonction des fonctions symétriques élémentaires des racines de \(P\): \[N_5 = \sum_{k=1}^3 x_k^2 +\sigma_1 -3 = \sigma_1^2-2\sigma_2+\sigma_1-3= -2-3=-5.\]

(2).
Soient trois complexes \((a,b,c) \in \mathbb{C} [3]\) de module \(1\) vérifiant \(a+b+c=1\). Montrer que l’un de ces trois complexes vaut \(1\).
Comme \(\lvert a \rvert =1\), \(\dfrac{1}{a} = \overline{a}\) et \(\overline{a+b+c} = \dfrac{1}{a} + \dfrac{1}{b} + \dfrac{1}{c} = 1\) d’où \(\sigma_1 = 1\) et \(\sigma_2 = ab + ac + bc = abc = \alpha\). Donc \((a,b,c)\) sont les racines du polynôme \[P = X^3 - X^2 + \alpha X - \alpha.\] Puisque \(P(1) = 0\), l’une des racines vaut \(1\).
Montrer qu’il n’existe pas de réels \((u,v,w)\) vérifiant : \[u+v+w=3 \quad\textrm{ et} \quad uv+vw+wu=6 .\]
\(u,v,w\) seraient racines du polynôme \(P(X)=X^3-3X^2+6X+c\) avec \(c = uvw \in \mathbb{R}\). Mais d’après le théorème de Rolle, \(P'\) posséderait alors deux racines réelles distinctes, ce qui est faux.

Calculs en algèbre linéaire

Symbole de Kronecker

Dans un corps \(\mathbb{K}\), pour deux indices \((i, j) \in [\kern-0.127em[ 1, n ]\kern-0.127em]^2\), on définit le symbole de Kronecker : \(\heartsuit\) \[\begin{aligned} \delta_{ij} = \begin{cases} 1_{\mathbb{K} } & \textrm{ si } i = j \newline 0_{\mathbb{K} } & \textrm{ si } i \neq j \end{cases}. \end{aligned}\]

Il apparaît souvent dans les calculs matriciels. La principale utilisation de ce symbole consiste à simplifier les sommes. Dans une somme \(S = \sum_{i \in I} \delta_{ip} a_i\), tous les termes de la somme sont nuls, sauf celui correspondant à l’indice \(i = p\) : \(S = a_p\).

\[\displaystyle{\sum_{i=1}^{n}} \delta_{i1} a_{ii} = a_{11},\] \[\displaystyle{\sum_{i=1}^{n}} \displaystyle{\sum_{j=1}^{n}} \delta_{ij} a_{ij} = \displaystyle{\sum_{i=1}^{n}} a_{ii}.\]

Utilisation des matrices \(E_{pq}\) en calcul matriciel

Dans ce paragraphe, nous allons voir quelques utilisations pratiques des matrices de la base canonique de \(\mathfrak{M}_{n}(\mathbb{\mathbb{K} })\).

(Matrices \(E_{pq}\)).
Pour \((p,q) \in [\kern-0.127em[ 1, n ]\kern-0.127em]^2\), on définit la matrice \[E_{pq} = \left( \delta_{ip} \delta_{iq}\right)_{1\leqslant i,j \leqslant n} \in \mathfrak{M}_{n}(\mathbb{\mathbb{K} }) .\] Les coefficients de cette matrice sont tous nuls, sauf le coefficient à la ligne \(p\) et à la colonne \(q\) qui vaut \(1_{\mathbb{K} }\).

Dans le chapitre [chap_matrice], on montre que la famille constituée de ces \(n^2\) matrices \((E_{pq})_{1\leqslant p,q \leqslant n}\) forme une base de \(\mathfrak{M}_{n}(\mathbb{\mathbb{K} })\). En particulier, toute matrice \(A = (a_{ij}) \in \mathfrak{M}_{n}(\mathbb{\mathbb{K} })\) se décompose en une combinaison linéaire de ces matrices : \[A = \displaystyle{\sum_{i=1}^{n}} \displaystyle{\sum_{j=1}^{n}} a_{ij} E_{ij}.\]

Le produit de deux matrices de la base canonique est simple à retenir : \(\heartsuit\) \[\begin{aligned} E_{pq} \times E_{kl} = \delta_{qk} E_{pl} \end{aligned}\]

Ces matrices fournissent des exemples simples en algèbre linéaire. Par exemple, si \(p \neq q\), \(E_{pq}^2 = E_{pq}E_{pq} = 0\) : la matrice \(E_{pq}\) est nilpotente d’indice \(2\). Pour \(n \geqslant 2\), l’anneau \(\mathfrak{M}_{n}(\mathbb{\mathbb{R} })\) n’est pas commutatif : \(E_{12} E_{21} = E_{11}\) alors que \(E_{21} E_{12} = E_{22}\)

Soit \(A = (a_{ij}) \in \mathfrak{M}_{n}(\mathbb{\mathbb{K} })\) et \((p,q) \in [\kern-0.127em[ 1, n ]\kern-0.127em]^2\). Calculer \(\mathop{\mathrm{Tr}}(AE_{pq})\). Décomposons la matrice \(A\) sur la base canonique : \[A = \displaystyle{\sum_{i=1}^{n}}\displaystyle{\sum_{j=1}^{n}} a_{ij} E_{ij}\] En utilisant la formule du produit \(E_{ij}E_{pq}\), la linéarité de la trace et la trace d’une matrice de la base canonique : \(\mathop{\mathrm{Tr}}(E_{iq}) = \delta_{iq}\), \[\mathop{\mathrm{Tr}}(AE_{pq}) = \displaystyle{\sum_{i=1}^{n}} \displaystyle{\sum_{j=1}^{n}} a_{ij} \mathop{\mathrm{Tr}}(E_{ij}E_{pq}) = \displaystyle{\sum_{i=1}^{n}}\displaystyle{\sum_{j=1}^{n}} a_{ij} \delta_{jp} \mathop{\mathrm{Tr}}(E_{iq}) = \displaystyle{\sum_{i=1}^{n}}\displaystyle{\sum_{j=1}^{n}} a_{ij}\delta_{jp}\delta_{iq} = a_{qp}.\]
Déterminer les formes linéaires \(\varphi: \mathfrak{M}_{n}(\mathbb{\mathbb{K} }) \mapsto \mathbb{K}\) vérifiant : \[\forall (A,B) \in \mathfrak{M}_{n}(\mathbb{\mathbb{K} }) ,~ \varphi(AB) = \varphi(BA).\] Soit \(\varphi\) une telle forme linéaire. Si l’on considère deux matrices quelconques \(A\) et \(B\), on a \(2n^2\) coefficients arbitraires et le calcul est pénible. Remarquons que si l’on connaît \(\varphi(E_{ij})\) pour tous \((i,j) \in [\kern-0.127em[ 1, n ]\kern-0.127em]\), \(\varphi\) étant linéaire, on connaîtra \(\varphi(A)\) pour toute matrice \(A\) : il suffit de décomposer \(A\) sur la base canonique. Utilisons cette idée :

Puisque \(\varphi\) vérifie la propriété pour deux matrices quelconques, elle la vérifie en particulier pour deux matrices de la base canonique \(E_{pq}\) et \(E_{kl}\), \(\varphi(E_{pq}E_{kl}) = \varphi(E_{kl}E_{pq})\) et on doit donc avoir : \[\forall \left(p,q,k,l\right) \in [\kern-0.127em[ 1, n ]\kern-0.127em]^4, \quad \delta_{qk}\varphi(E_{pl}) = \delta_{lp} \varphi(E_{kq})\] Soient \(\left(i, j\right) \in [\kern-0.127em[ 1, n ]\kern-0.127em]^2\), avec \(i \neq j\).

  • En prenant \(p=i\), \(l=j\), \(q=k=i\), on trouve que \(\varphi(E_{ij}) = \delta_{ji}\varphi(E_{ii}) = 0\).

  • En prenant \(p=l=i\) et \(q=k=j\), on trouve que \(\varphi(E_{ii}) = \varphi(E_{jj})\), c’est-à-dire que \(\varphi\) prend la même valeur sur toutes les matrices \(E_{ii}\).

En notant \(\alpha = \varphi(E_{11}) = \dots = \varphi(E_{nn})\) cette valeur commune, on a donc \(\varphi(E_{ij}) = \alpha \delta_{ij}\). Pour une matrice \(A\) quelconque, \[\varphi(A) = \varphi(\displaystyle{\sum_{i=1}^{n}}\displaystyle{\sum_{j=1}^{n}} a_{ij} E_{ij}) = \displaystyle{\sum_{i=1}^{n}} \displaystyle{\sum_{j=1}^{n}} a_{ij} \varphi(E_{ij}) = \displaystyle{\sum_{i=1}^{n}} a_{ii} \alpha \delta_{ij} = \alpha \displaystyle{\sum_{i=1}^{n}} a_{ii} = \alpha \mathop{\mathrm{Tr}}(A)\] Nous avons montré qu’il existait \(\alpha \in \mathbb{K}\) tel que \(\varphi= \alpha \mathop{\mathrm{Tr}}\). Réciproquement, pour tout \(\alpha \in \mathbb{K}\), en posant \(\varphi= \alpha \mathop{\mathrm{Tr}}\), \(\varphi\) vérifie la propriété.
(1).
Soit deux matrices \(A, B \in \mathfrak{M}_{n}(\mathbb{K})\). On suppose que \[\forall X \in \mathfrak{M}_{n}(\mathbb{K}) \quad\mathop{\mathrm{Tr}}(AX)=\mathop{\mathrm{Tr}}(BX) .\] Montrer que \(A=B\).
Si \(A=(a_{ij})\) et \(X=(x_{ij})\), on calcule \[\mathop{\mathrm{Tr}}(AX) = \sum_{i=1}^n\sum_{k=1}^n a_{ik}x_{ki} .\] En prenant \(X= E_{pq}\), \(x_{ki}=\delta_{kp}\delta_{iq}\), \(\mathop{\mathrm{Tr}}(AX)= a_{qp}\) et \(\forall (q,p)\in \llbracket 1,n\rrbracket\), \(a_{qp}=b_{qp}\) d’où \(A=B\).
(2).
Soit une sous-algèbre \(\mathcal{A}\) de l’algèbre \(\mathfrak{M}_{n}(\mathbb{\mathbb{K} })\). On suppose que \[\forall M \in \mathfrak{M}_{n}(\mathbb{\mathbb{K} }) , \quad M^2 \in \mathcal{A} \Rightarrow M \in \mathcal{A}\] Montrer que \(\mathcal{A} = \mathfrak{M}_{n}(\mathbb{\mathbb{R} })\).
Supposons que \(\mathcal{A}\) soit une sous-algèbre de \(\mathfrak{M}_{n}(\mathbb{\mathbb{K} })\) vérifiant \(\forall M \in \mathfrak{M}_{n}(\mathbb{\mathbb{K} })\), \(M^2 \in \mathcal{A} \Rightarrow M \in \mathcal{A}\). Il suffit de montrer que toute matrice \(E_{ij}\) de la base canonique appartient à \(\mathcal{A}\). Comme \(\mathcal{A}\) est une sous-algèbre de \(\mathfrak{M}_{n}(\mathbb{\mathbb{K} })\), \(0_{\mathfrak{M}_{n}(\mathbb{\mathbb{K} }) } \in \mathcal{A}\). Or si \((i, j) \in [\kern-0.127em[ 1, n ]\kern-0.127em]^2\), \(E_{ij}^2 = E_{ij}E_{ij} = \delta_{ij} E_{ij}\). Par conséquent, si \(i \neq j\), \(E_{ij}^2 \in \mathcal{A}\) et \(E_{ij} \in \mathcal{A}\). Soit maintenant \(i \in [\kern-0.127em[ 1, n ]\kern-0.127em]\). Soit \(j \neq i\). On sait que \(E_{ij}, E_{ji} \in \mathcal{A}\) et, puisque \(\mathcal{A}\) est une sous-algèbre de \(\mathfrak{M}_{n}(\mathbb{\mathbb{K} })\), le produit \(E_{ij}E_{ji} = E_{ii}\) est encore dans \(\mathcal{A}\). Comme \(\mathcal{A}\) contient toutes les matrices de la base canonique et que c’est un sous-espace vectoriel de \(\mathfrak{M}_{n}(\mathbb{\mathbb{K} })\), \(\mathcal{A} = \mathfrak{M}_{n}(\mathbb{\mathbb{K} })\).

Regardons ce que donne la multiplication à gauche et à droite d’une matrice par une matrice de la base canonique :

Dessins

  1. Lorsqu’on pose la multiplication \(E_{pq}A\), on remarque que toutes les lignes du produit sont nulles sauf la ligne \(p\) où l’on retrouve les coefficients de la ligne \(L_q\) de la matrice \(A\).

  2. Lorsqu’on pose la multiplication \(AE_{pq}\), toutes les colonnes du produit sont nulles sauf la colonne \(q\) où l’on retrouve les coefficients de la colonne \(C_p\) de la matrice \(A\).

Un premier exemple illustre l’utilisation de ces matrices :

Déterminer les matrices \(A \in \mathfrak{M}_{n}(\mathbb{\mathbb{K} })\) qui commutent avec toutes les matrices de \(\mathfrak{M}_{n}(\mathbb{\mathbb{K} })\). Soit \(A\) une telle matrice. Puisqu’elle commute avec toutes les matrices, elle commute en particulier avec les matrices de la base canonique : \[\forall (p, q) \in [\kern-0.127em[ 1, n ]\kern-0.127em]^2,\quad E_{pq}A = AE_{pq}.\] Mais on a calculé \(E_{pq} A\) et \(AE_{pq}\) ci-dessus et on en déduit que \(\forall j \neq q\), \(a_{qj} = 0\) et, en examinant le coefficient à la ligne \(p\) et à la colonne \(q\) de ces deux produits, on obtient : \(\forall p, q \in [\kern-0.127em[ 1, n ]\kern-0.127em]\), \(a_{qq} = a_{pp}\). En notant \(\alpha = a_{11} = \dots = a_{nn}\), on a nécessairement \(A = \alpha I_n\) : \(A\) est une matrice scalaire. Réciproquement, toute matrice scalaire commute avec toutes les matrices.
(1).
Déterminer les matrices \(A \in \mathfrak{M}_{n}(\mathbb{\mathbb{R} })\) qui commutent avec toutes les matrices symétriques.
\(A\) doit commuter avec toutes les matrices symétriques \(E_{pp}\) et on trouve que \(A\) doit nécessairement être diagonale. Ensuite, \(A\) doit commuter avec toutes les matrices \(S_{ij} = E_{ij} + E_{ji}\) et, en effectuant le calcul, on trouve que \(A\) doit être une matrice scalaire.

Calcul de déterminants

Faire apparaître des zéros

C’est, peu ou prou, la méthode de Gauss. La méthode est plus souple : on peut travailler sur les lignes et sur les colonnes.

Calculer \[D = \begin{vmatrix} -8 & 0 & 3 & 2 \\ -1 & 8 & -10 & -7 \\ -2 & -7 & 4 & -9 \newline -5 & -4 & -2 & 1 \end{vmatrix}.\]
On est en présence d’une matrice à coefficients entiers. On utilise la méthode du pivot de Gauss. On utilise des petits pivots (en valeur absolue) pour éviter les quotients. \(\begin{array}{|cccc|c|cccc|} -8 & 0 & 3 & 2 & L_1 \longleftarrow L_1 - 2 L_4 & 2 & 8 & 7 & 0 \\ -1 & 8 & -10 & -7 & L_2 \longleftarrow L_2 + 7 L_4 & -36 & -20 & -24 & 0\\ -2 & -7 & 4 & -9 & L_3 \longleftarrow L_3 + 9 L_4 & -47 & -43 & -14 & 0\\ -5 & -4 & -2 & 1 && -5 & -4 & -2 & 1 \end{array}\).

On fait apparaître un pivot égal à \(1\) : \(C_2 \longleftarrow C_2 - C_3\).

\(D = \begin{vmatrix} 2 & 8 & 7 \\ -36 & -20 & -24 \\ -47 & -43 & -14 \end{vmatrix} = \begin{array}{|ccc|c|ccc|} 2 & 1 & 7 && 2 & 1 & 7\\ -36 & 4 & -24 &L_2 \longleftarrow L_2 - 4 L_1 & -44 & 0 & -52\newline -47 & -29 & -14 &L_3 \longleftarrow L_3 + 29 L_1 &7 &0& 175 \end{array}\).

D’où \(D = -(-44 \times 189 - (-52) \times 11) = 7744\).

Factorisation

C’est l’hygiène du calcul de déterminants. Repérer un facteur commun sur une ligne ou une colonne et mettre en facteur simplifie le travail. En particulier lorsque la somme des lignes est constante, un bon réflexe est d’additionner toutes les lignes dans première, de mettre en facteur (on n’a plus que des 1 sur la première ligne) puis on peut soustraire la première colonne de toutes les autres...

Soit \(a,b,c,x\) quatre complexes. Factoriser \[D = \begin{vmatrix} x & a & b & c \\ a & x & c & b \\ b & c & x & a \newline c & b & a & x \end{vmatrix}.\]
En additionnant toutes les lignes dans la première, on fait apparaître \(x+a+b+c\) : \[D = \begin{vmatrix} x+a+b+c & x+a+b+c & x+a+b+c & x+a+b+c \\ a & x & c & b \\ b & c & x & a \\ c & b & a & x \end{vmatrix} = (x+a+b+c) \begin{vmatrix} 1 & 1 & 1 & 1 \\ a & x & c & b \\ b & c & x & a \\ c & b & a & x \end{vmatrix} .\] La présence des \(1\) sur la première ligne incite à soustraire par exemple la dernière colonne dans les autres : \[D = (x+a+b+c) \begin{vmatrix} 0 & 0 & 0 & 1 \\ a-b & x-b & c-b & b\\ b-a & c-a & x-a & a\\ c-x & b-x & a-x & x \end{vmatrix} = (x+a+b+c) \times (-1) \times \begin{vmatrix} a-b & x-b & c-b \\ b-a & c-a & x-a \\ c-x & b-x & a-x \end{vmatrix}\] On additionne la deuxième ligne dans la première : \[D = -(x+a+b+c) \begin{vmatrix} 0 & x-b+c-a & x-a+c-b \\ b-a & c-a & x-a \\ c-x & b-x & a-x \end{vmatrix} = -(x+a+b+c)(x-a-b+c) \begin{vmatrix} 0 & 1 & 1 \\ b-a & c-a & x-a \\ c-x & b-x & a-x \end{vmatrix}\] On soustrait la troisième colonne dans la deuxième, on développe ensuite suivant la première ligne : \[\begin{aligned} D &= -(x+a+b+c)(x-a-b+c) \begin{vmatrix} 0 & 0 & 1 \\ b-a & c-x & x-a \\ c-x & b-a & a-x \end{vmatrix} \\ &= -(x+a+b+c)(x-a-b+c) \begin{vmatrix} b-a & c-x \\ c-x & b-a \end{vmatrix} = (x+a+b+c)(x-a-b+c)\left[ (c-x)^2-(b-a)^2\right] \newline &= (x+a+b+c)(x-a-b+c)(x-c-b+a)(x-c+b-a).\end{aligned}\]
Soit \(\alpha_1,\alpha_2,\alpha_3\) trois réels. Calculer \[D = \begin{vmatrix} 1 & \cos\alpha_1 & \tan {\scriptstyle\alpha_1\over\scriptstyle 2} \\ 1 & \cos\alpha_2 & \tan {\scriptstyle\alpha_2\over\scriptstyle 2} \newline 1 & \cos\alpha_3 & \tan {\scriptstyle\alpha_3\over\scriptstyle 2} \end{vmatrix}.\] Démontrer que si \(\alpha_1+\alpha_2+\alpha_3 = \pi\) alors \(D=0\).
Soit \(t_k = \tan {\scriptstyle\alpha_k\over\scriptstyle 2}\), on a \[\begin{aligned} D &= \begin{vmatrix} 1 & \dfrac{1-t_1^2}{1+t_1^2} & t_1 \\ 1 & \dfrac{1-t_2^2}{1+t_2^2} & t_2 \\ 1 & \dfrac{1-t_3^2}{1+t_3^2} & t_3 \end{vmatrix} = \dfrac{1}{(1+t_1^2)(1+t_2^2)(1+t_3^2)} \begin{vmatrix} 1+t_1^2 & 1-t_1^2 & t_1(1+t_1^2) \\ 1+t_2^2 & 1-t_2^2 & t_2(1+t_2^2) \\ 1+t_3^2 & 1-t_3^2 & t_3(1+t_3^2) \end{vmatrix}\\ &= \dfrac{1}{(1+t_1^2)(1+t_2^2)(1+t_3^2)} \begin{vmatrix} 1+t_1^2 & 2 & t_1+t_1^3 \\ 1+t_2^2 & 2 & t_2+t_2^3 \\ 1+t_3^2 & 2 & t_3+t_3^3 \end{vmatrix}\end{aligned}\] en soustrayant la première colonne dans la deuxième. Maintenant on soustrait la première ligne dans les deux autres : \[D = \dfrac{1}{(1+t_1^2)(1+t_2^2)(1+t_3^2)} \begin{vmatrix} 1+t_1^2 & 2 & t_1+t_1^3 \\ t_2^2 - t_1^2 & 0 & t_2-t_1+t_2^3-t_1^3 \\ t_3^2 - t_1^2 & 0 & t_3-t_1+t_3^3-t_1^3 \end{vmatrix} = \dfrac{(t_2-t_1)(t_3-t_1)}{(1+t_1^2)(1+t_2^2)(1+t_3^2)} \begin{vmatrix} 1+t_1^2 & 2 & t_1+t_1^3 \\ t_2+ t_1 & 0 & 1+t_2^2+t_1^2+t_1t_2 \\ t_3+ t_1 & 0 & 1+t_3^2+t_1^2+t_1t_3 \end{vmatrix}\] On développe par rapport à la deuxième colonne, \[\begin{aligned} D &= -2\dfrac{(t_2-t_1)(t_3-t_1)}{(1+t_1^2)(1+t_2^2)(1+t_3^2)} \begin{vmatrix} t_2+ t_1 & 1+t_2^2+t_1^2+t_1t_2 \\ t_3+ t_1 & 1+t_3^2+t_1^2+t_1t_3 \end{vmatrix} = -2\dfrac{(t_2-t_1)(t_3-t_1)}{(1+t_1^2)(1+t_2^2)(1+t_3^2)} \begin{vmatrix} t_2+ t_1 & 1+t_2^2+t_1^2+t_1t_2 \\ t_3- t_2 & t_3^2-t_2^2+t_1(t_3-t_2) \end{vmatrix} \\ & = -2\dfrac{(t_2-t_1)(t_3-t_1)(t_3- t_2)}{(1+t_1^2)(1+t_2^2)(1+t_3^2)} \begin{vmatrix} t_2+ t_1 & 1+t_2^2+t_1^2+t_1t_2 \\ 1 & t_3+t_2+t_1 \end{vmatrix} = 2\dfrac{(t_2-t_1)(t_1-t_3)(t_3- t_2)}{(1+t_1^2)(1+t_2^2)(1+t_3^2)} \left[(t_2+ t_1)(t_3+t_2+t_1) - (1+t_2^2+t_1^2+t_1t_2)\right] \newline &= 2\dfrac{(t_2-t_1)(t_1-t_3)(t_3- t_2)}{(1+t_1^2)(1+t_2^2)(1+t_3^2)} (t_1t_2 + t_2t_3 + t_3t_1 - 1) \end{aligned}\]

Comme \(\tan(\vartheta_1+\vartheta_2+\vartheta_3) = \dfrac{\tan\vartheta_1+\tan\vartheta_2+\tan\vartheta_3}{ 1-\tan\vartheta_1\tan\vartheta_2-\tan\vartheta_2\tan\vartheta_3-\tan\vartheta_3\ tan\vartheta_1}\), en prenant \(\vartheta_k = {\scriptstyle\alpha_k\over\scriptstyle 2}\), comme \(\alpha_1+\alpha_2+\alpha_3 = \pi\), on en déduit que le dénominateur \(1-\tan\vartheta_1\tan\vartheta_2-\tan\vartheta_2\tan\vartheta_3-\tan\vartheta_3 \tan\vartheta_1\) s’annule, autrement dit \(t_1t_2 + t_2t_3 + t_3t_1 - 1 = 0\) et par suite \(D=0\).

Cela signifie que les points de paramètres \(\alpha_k\) avec \(\alpha_1+\alpha_2+\alpha_3 = \pi\) sont alignés sur la courbe \(x = \cos t ; y = \tan {\scriptstyle t\over\scriptstyle 2}\).

Techniques polynomiales

Lorsque les coefficients sont des polynômes, le déterminant est lui-même un polynôme.

(2).
Soit \(a_0, a_1, \ldots, a_{n-1} \in \mathbb{C}^n\), soit \(x\in\mathbb{C}\) .
Calculer \[D(x) = \begin{vmatrix} -x & 0 & \dots & 0 & -a_0 \\ 1 & -x & \dots & 0 & -a_1 \\ 0 & 1 & \ddots & \vdots & -a_2 \\ \vdots & \vdots & \ddots & \ddots & \vdots \\ 0 & \dots & 0 & 1 & -a_{n-1}-x \newline \end{vmatrix}.\]
On développe par rapport à la dernière colonne : \[\begin{aligned} D(x) &= (-1)^{n-1}\times(-a_0) \begin{vmatrix} 1 & -x & 0 & \dots & 0 \\ 0 & 1 & \ddots & \vdots & 0 \\ 0 & 0 & 1 & \vdots & 0 \\ \vdots & \ddots & \ddots & 1 & \vdots \\ 0 &\dots & \dots & 0 & 1 \end{vmatrix} + (-1)^{n}\times(-a_1) \begin{vmatrix} -x & 0 & \dots & \dots & 0 \\ 0 & 1 & -x & \vdots & 0 \\ 0 & 0 & 1 & \vdots & 0 \\ \vdots & \ddots & \ddots & 0 & \vdots \\ 0 &\dots & \dots & 0 & 1 \end{vmatrix} \\ & + \ldots + (-1)^{n+k-1}\times(-a_k) \begin{vmatrix} -x & 0 & \dots & \dots & 0 \\ \vdots & \ddots & \ddots & \vdots & \vdots \\ 0 & \dots & -x & 0 & \vdots \\ \vdots & \ddots & \ddots & 1 & \vdots \\ 0 &\dots & \dots & 0 & 1 \end{vmatrix} + \ldots + (-a_{n-1}-x) \begin{vmatrix} -x & 0 & \dots & \dots & 0 \\ 1 & -x & \ddots & \vdots & 0 \\ 0 & 1 & -x & \vdots & 0 \\ \vdots & \ddots & \ddots & -x & 1 \\ 0 &\dots & \dots & 1 & -x \end{vmatrix}\end{aligned}\] En développant les déterminants par blocs (triangulaires) \[\begin{aligned} D(x) &= (-1)^{n-1}\times(-a_0)\times1 + (-1)^{n}\times(-a_1)\times(-x) + \ldots + (-1)^{n+k-1}\times(-a_k)\times(-x)^k + \ldots + (-1)^{n+n-2-1}\times(-a_{n-2})\times(-x)^{n-2} \\ &+ (-1)^{n+n-1-1}\times(-a_{n-1}-x)\times(-x)^{n-1}\end{aligned}\] Puis, en effectuant les calculs, \[\begin{aligned} D(x) &= (-1)^{n}a_0 + (-1)^{n}a_1x + \ldots + (-1)^{n}a_kx^k + \ldots + (-1)^{n}a_{n-2}x^{n-2} + (-1)^{n}(a_{n-1}+x)x^{n-1}\newline &= (-1)^{n}\left( a_0 + a_1x + \ldots + a_{n-1}x^{n-1}+x^n\right). \end{aligned}\]

On peut reprendre un exercice précédent en simplifiant les calculs ... à condition d’avoir le bon coup d’œil et de connaître ses déterminants de Vandermonde :

(2).
Calculer \[\Delta = \begin{vmatrix} 1+t_1^2 & 1-t_1^2 & t_1(1+t_1^2) \\ 1+t_2^2 & 1-t_2^2 & t_2(1+t_2^2) \newline 1+t_3^2 & 1-t_3^2 & t_3(1+t_3^2) \end{vmatrix}.\]
Comme précédemment, \[\Delta = \begin{vmatrix} 1+t_1^2 & 2 & t_1(1+t_1^2) \\ 1+t_2^2 & 2 & t_2(1+t_2^2) \\ 1+t_3^2 & 2 & t_3(1+t_3^2) \end{vmatrix} = 2 \begin{vmatrix} 1+t_1^2 & 1 & t_1(1+t_1^2) \\ 1+t_2^2 & 1 & t_2(1+t_2^2) \\ 1+t_3^2 & 1 & t_3(1+t_3^2) \end{vmatrix} = 2 \begin{vmatrix} t_1^2 & 1 & t_1(1+t_1^2) \\ t_2^2 & 1 & t_2(1+t_2^2) \\ t_3^2 & 1 & t_3(1+t_3^2) \end{vmatrix}.\] En utilisant la multilinéarité du déterminant : \[\Delta = 2 \begin{vmatrix} t_1^2 & 1 & t_1 \\ t_2^2 & 1 & t_2 \\ t_3^2 & 1 & t_3 \end{vmatrix} +2 \begin{vmatrix} t_1^2 & 1 & t_1^3 \\ t_2^2 & 1 & t_2^3 \\ t_3^2 & 1 & t_3^3 \end{vmatrix} .\]

Le premier déterminant est le déterminant de Vandermonde \(V_3(t_1,t_2,t_3) = (t_2-t_1)(t_3-t_1)(t_3-t_2)\) et le second est le coefficient de \(x\) dans \(\begin{vmatrix} 1 & x & x^2 & x^3 \\ 1 & t_1 & t_1^2 & t_1^3 \\ 1 & t_2 & t_2^2 & t_2^3 \\ 1 & t_3 & t_3^2 & t_3^3 \end{vmatrix} = V_4(x,t_1,t_2,t_3)\).
Or \(V_4(x,t_1,t_2,t_3) = (x-t_1)(x-t_2)(x-t_3)V_3(t_1,t_2,t_3)(t_1t_2+t_2t_3+t_3t_1)\). Ainsi le coefficient de \(x\) égale \(-(t_1t_2+t_2t_3+t_3t_1)V_3(t_1,t_2,t_3)\).

Finalement, \(\Delta = -2V_3(t_1,t_2,t_3)(t_1t_2+t_2t_3+t_3t_1-1)\).

L’avantage d’être un polynôme, c’est qu’il est déterminé par un nombre fini de valeurs.

(2).
  1. Soit \(a,b\) deux réels distincts, \(x_1,\ldots,x_n\), \(n\) nombres réels.
    En ajoutant \(x\) à chacun des éléments, calculer \[D_{a,b} = \begin{vmatrix} x_1 & b & \dots & b & b \\ a & x_2 & \dots & \dots & b \\ a & a & x_3 & \ddots & \vdots \\ \vdots & \ddots & \ddots & \ddots & b \\ a & \dots & \dots & a & x_n \\ \end{vmatrix}.\]

  2. Soit \(a\) un réel, \(x_1,\ldots,x_n\), \(n\) nombres réels.
    Calculer \[D_{a} = \begin{vmatrix} x_1 & a & \dots & a & a \\ a & x_2 & \dots & \dots & a \\ a & a & x_3 & \ddots & \vdots \\ \vdots & \ddots & \ddots & \ddots & a \\ a & \dots & \dots & a & x_n \newline \end{vmatrix}.\]

  1. Cet exercice illustre l’adage : Plus une égalité comporte de variables, plus elle est facile à démontrer !
    Ici on cherche à calculer \(D_{a,b} = D_{a,b}(0)\) avec \[D_{a,b}(x) = \begin{vmatrix} x_1 + x & b + x & \dots & b + x & b + x \\ a + x & x_2 + x & \dots & \dots & b + x \\ a + x & a + x & x_3 + x & \ddots & \vdots \\ \vdots & \ddots & \ddots & \ddots & b + x\\ a + x & \dots & \dots & a + x & x_n + x\\ \end{vmatrix}.\] \(D_{a,b}(x)\) est un polynôme en \(x\).
    En retranchant la première colonne à toutes les autres puis en développant par rapport à la première colonne (ou en dérivant deux fois par rapport à \(x\) le déterminant \(D_{a,b}(x)\)), on voit que \(D_{a,b}(x)\) est un polynôme en \(x\) de degré inférieur ou égal à \(1\), c’est-à-dire une fonction affine. \(D_{a,b}(x) = \alpha x + \beta\).

    Maintenant, il suffit de déterminer la valeur de \(D_{a,b}(x)\) en deux points distincts. Par exemple en \(-a\) et \(-b\) on voit alors qu’on a à calculer le déterminant d’une matrice triangulaire. \[D_{a,b}(-a) = \prod_{k=1}^n(x_k-a)\qquad \textrm{ et }\qquad D_{a,b}(-b) = \prod_{k=1}^n(x_k-b).\] D’où \[\alpha = \dfrac{D_{a,b}(-a) - D_{a,b}(-b)}{b-a} = \dfrac{1}{b-a}\left( \prod_{k=1}^n(x_k-a) - \prod_{k=1}^n(x_k-b)\right).\] En posant \(\Pi_t= \displaystyle\prod_{k=1}^n(x_k-t)\), la valeur qui nous intéresse est \(\beta = D_{a,b}(0) = \Pi_a + a\dfrac{\Pi_b - \Pi_a}{b-a}\).

  2. \(a\) étant fixé, la fonction \(D~: b \longmapsto D_{a,b}\) est continue puisque c’est un polynôme en \(b\).
    Donc \(D_a = D(a) = \displaystyle\lim_{b\to a} D(b) = \Pi_a + a\Pi_a' = \displaystyle\prod_{k=1}^n(x_k-a) - \sum_{i=1}^n \prod_{\substack{1\leqslant k\leqslant n\newlinek\neq i}} (x_k-a)\).

Un exercice déjà vu :

(2).
Soit \(a,b,c,x\) quatre complexes. Factoriser \[P_{a,b,c}(x) = \begin{vmatrix} x & a & b & c \\ a & x & c & b \\ b & c & x & a \newline c & b & a & x \end{vmatrix}.\]
\(P_{a,b,c}\) est un polynôme de degré inférieur ou égal à \(4\). Le coefficient de \(x^4\) ne peut être obtenu qu’en prenant les \(x\) de la diagonale principale. Donc ce coefficient égale \(1\).

En additionnant toutes les lignes dans la première, on trouve \(x+a+b+c\) : \[P_{a,b,c}(x) = \begin{vmatrix} x+a+b+c & x+a+b+c & x+a+b+c & x+a+b+c \\ a & x & c & b \\ b & c & x & a \\ c & b & a & x \end{vmatrix} = (x+a+b+c) \begin{vmatrix} 1 & 1 & 1 & 1 \\ a & x & c & b \\ b & c & x & a \newline c & b & a & x \end{vmatrix} .\] Autrement dit, le polynôme \(X+a+b+c\) divise \(P_{a,b,c}(X)\).

Cette fois nous allons multiplier la deuxième et la quatrième ligne par \(-1\). Nous additionnons le tout dans la première ligne et nous pouvons factoriser \(x - a + b - c\). Cette fois, c’est \(X-a+b-c\) qui divise \(P_{a,b,c}(X)\).

En multipliant la deuxième et la troisième ligne par \(-1\), on trouve \(X-a-b+c\) divise \(P_{a,b,c}(X)\) et en multipliant la troisième et la quatrième ligne par \(-1\), on trouve que \(X+a-b-c\) divise \(P_{a,b,c}(X)\).

Lorsque les quatre nombres \(a+b+c\), \(-a+b-c\), \(-a-b+c\), \(a-b-c\) sont distincts deux à deux, alors les quatre polynômes \(X+a+b+c\) sont premiers entre eux, donc leur produit divise \(P_{a,b,c}(X)\). Comme \(P_{a,b,c}(X)\) et \((X+a+b+c)(X-a+b-c)(X-a-b+c)(X+a-b-c)\) sont deux polynômes de degré \(4\), que le deuxième divise le premier et qu’ils sont tous deux unitaires, on a \[\forall\, x\in\mathbb C,\,P_{a,b,c}(x) = (x+a+b+c)(x-a+b-c)(x-a-b+c)(x+a-b-c).\]

Dans le cas où au moins deux des nombres \(a+b+c;\,-a+b-c;\,-a-b+c\) et \(a-b-c\) sont égaux, on remplace \(b\) par \(b' = b+h\) et \(c\) par \(c' = c+2h\). La fonction \(h\longmapsto P_{a,b',c'}(x) = (x+a+b'+c')(x-a+b'-c')(x-a-b'+c')(x+a-b'-c')\) est un polynôme en \(h\) donc une application continue, qui est nulle d’après ce qui précède pour \(h>0\) assez petit pour que les \(a+b'+c'\), \(-a+b'-c'\), \(-a-b'+c'\) et \(a-b'-c'\) soient tous distincts. (Il suffit de prendre \(h\) plus petit que la moitié du plus petit des nombres strictement positifs parmi \(\vert a\pm b\vert\), \(\vert a\pm c\vert\) et \(\vert b\pm c\vert\).) Donc en faisant tendre \(h\) vers zéro, on a bien \(P_{a,b,c}(x) - (x+a+b+c)(x-a+b-c)(x-a-b+c)(x+a-b-c) = 0\), ce qu’il fallait démontrer.

Remarque : Il faut parfois se torturer les méninges pour éviter des calculs pas forcément méchants. C’était surtout pour voir la méthode. Méthode qui est beaucoup plus efficace avec les polynômes à plusieurs indéterminées (qui ne sont pas au programme). En effet, plus il y a de variables...

Dérivation

Un déterminant peut être une fonction d’une (ou plusieurs) variable(s). Il faut savoir la dériver.

(2).
Soit \(a,b,c,d\) quatre réels. On pose \(\Delta(d) = \begin{vmatrix} \cos(a+d) & \sin(a+d) & \cos(b-c) \\ \cos(b+d) & \sin(b+d) & \cos(c-a) \newline \cos(c+d) & \sin(c+d) & \cos(a-b) \end{vmatrix}\).
  1. Calculer \(\Delta(b-c-a)\).

  2. Calculer la dérivée \(\Delta'(d)\).

  3. Conclure.

  1. On a \(a+d = b-c;\;b+d = 2b-c-a;\;c+d = b-a\). Donc \[\begin{aligned} \Delta(b-c-a) &= \begin{vmatrix} \cos(b-c) & \sin(b-c) & \cos(b-c) \\ \cos(2b-c-a) & \sin(2b-c-a) & \cos(c-a) \\ \cos(b-a) & \sin(b-a) & \cos(a-b) \end{vmatrix}\\ &= \begin{vmatrix} 0 & \sin(b-c) & \cos(b-c) \\ \cos(2b-c-a) - \cos(c-a) & \sin(2b-c-a) & \cos(c-a) \\ 0 & \sin(b-a) & \cos(a-b) \end{vmatrix} \\ &= -(\cos(2b-c-a) - \cos(c-a)) \begin{vmatrix} \sin(b-c) & \cos(b-c) \\ \sin(b-a) & \cos(a-b) \end{vmatrix}\\ &= (\cos(c-a) - \cos(2b-c-a))(\sin(b-c)\cos(b-a)-\cos(b-c)\sin(b-a)) \\ &= (\cos(c-a) - \cos(2b-c-a))\sin(b-c-(b-a)) = (\cos(c-a) - \cos(2b-c-a))\sin(a-c)\\ &= \dfrac12\sin(2a-2c) - \dfrac12\left( \sin(2b-2c) + \sin(2a-2b)\right) = \dfrac12\left(\sin(2a-2c) + \sin(2c-2b) + \sin(2b-2a)\right).\end{aligned}\]

  2. En notant \(C_k(d)\) la \(k\)-ième colonne. On a (voir proposition [derivee_determinant] p. ) \[\Delta'(d) = \left\vert C'_1(d) C_2(d) C_3(d) \right\vert + \left\vert C_1(d) C'_2(d) C_3(d) \right\vert + \left\vert C_1(d) C_2(d) C'_3(d) \right\vert.\] (On aurait bien sûr un résultat semblable avec les lignes). On a donc \[\begin{aligned} \Delta'(d) &= - \begin{vmatrix} \sin(a+d) & \sin(a+d) & \cos(b-c) \\ \sin(b+d) & \sin(b+d) & \cos(c-a) \\ \sin(c+d) & \sin(c+d) & \cos(a-b) \end{vmatrix} + \begin{vmatrix} \cos(a+d) & \cos(a+d) & \cos(b-c) \\ \cos(b+d) & \cos(b+d) & \cos(c-a) \\ \cos(c+d) & \cos(c+d) & \cos(a-b) \end{vmatrix} + \begin{vmatrix} \cos(a+d) & \sin(a+d) & 0 \\ \cos(b+d) & \sin(b+d) & 0 \\ \cos(c+d) & \sin(c+d) & 0 \end{vmatrix} \\ &= 0+0+0 \end{aligned}\]

  3. Conclusion : \(\Delta(d)\) ne dépend pas de \(d\) et \(\forall d\in\mathbb R, \Delta(d) = \dfrac12\left(\sin(2a-2c) + \sin(2c-2b) + \sin(2b-2a)\right)\). Plus il y a de variables...

Matrices

Les déterminants sont utiles pour les matrices, pourquoi pas l’inverse ?

Soit \(a,b,c,d\in\mathbb C\). Calculer \[\begin{vmatrix} b+c+d & a+c+d & a+b+d & a+b+c \\ b^2+c^2+d^2 & a^2+c^2+d^2 & a^2+b^2+d^2 & a^2+b^2+c^2 \\ b^3+c^3+d^3 & a^3+c^3+d^3 & a^3+b^3+d^3 & a^3+b^3+c^3 \newline b^4+c^4+d^4 & a^4+c^4+d^4 & a^4+b^4+d^4 & a^4+b^4+c^4 \end{vmatrix}.\]
On a \[\begin{pmatrix} b+c+d & a+c+d & a+b+d & a+b+c \\ b^2+c^2+d^2 & a^2+c^2+d^2 & a^2+b^2+d^2 & a^2+b^2+c^2 \\ b^3+c^3+d^3 & a^3+c^3+d^3 & a^3+b^3+d^3 & a^3+b^3+c^3 \\ b^4+c^4+d^4 & a^4+c^4+d^4 & a^4+b^4+d^4 & a^4+b^4+c^4 \end{pmatrix} = \begin{pmatrix} a & b & c & d \\ a^2 & b^2 & c^2 & d^2 \\ a^3 & b^3 & c^3 & d^3 \\ a^4 & b^4 & c^4 & d^4 \end{pmatrix} \times \begin{pmatrix} 0 & 1 & 1 & 1 \\ 1 & 0 & 1 & 1 \\ 1 & 1 & 0 & 1 \\ 1 & 1 & 1 & 0 \end{pmatrix}.\] Á l’aide d’un déterminant de Vandermonde : \[\begin{vmatrix} a & b & c & d \\ a^2 & b^2 & c^2 & d^2 \\ a^3 & b^3 & c^3 & d^3 \\ a^4 & b^4 & c^4 & d^4 \end{vmatrix} = abcd \begin{vmatrix} 1 & 1 & 1 & 1 \\ a & b & c & d \\ a^2 & b^2 & c^2 & d^2 \\ a^3 & b^3 & c^3 & d^3 \end{vmatrix} = abcd(d-c)(d-b)(d-a)(c-b)(c-a)(b-a).\] En additionnant dans la première ligne : \[\begin{vmatrix} 0 & 1 & 1 & 1 \\ 1 & 0 & 1 & 1 \\ 1 & 1 & 0 & 1 \\ 1 & 1 & 1 & 0 \end{vmatrix} = 3 \begin{vmatrix} 1 & 1 & 1 & 1 \\ 1 & 0 & 1 & 1 \\ 1 & 1 & 0 & 1 \\ 1 & 1 & 1 & 0 \end{vmatrix} = 3 \begin{vmatrix} 0 & 1 & 1 & 1 \\ 0 & 0 & 1 & 1 \\ 0 & 1 & 0 & 1 \\ 1 & 1 & 1 & 0 \end{vmatrix} = -3\begin{vmatrix} 1 & 1 & 1 \\ 0 & 1 & 1 \\ 1 & 0 & 1 \end{vmatrix} = -3\begin{vmatrix} 1 & 1 & 0 \\ 0 & 1 & 1 \\ 1 & 0 & 0 \end{vmatrix} = 3\begin{vmatrix} 1 & 1 \\ 1 & 0 \end{vmatrix} = -3.\] Donc \[\begin{vmatrix} b+c+d & a+c+d & a+b+d & a+b+c \\ b^2+c^2+d^2 & a^2+c^2+d^2 & a^2+b^2+d^2 & a^2+b^2+c^2 \\ b^3+c^3+d^3 & a^3+c^3+d^3 & a^3+b^3+d^3 & a^3+b^3+c^3 \newline b^4+c^4+d^4 & a^4+c^4+d^4 & a^4+b^4+d^4 & a^4+b^4+c^4 \end{vmatrix} = -3abcd(d-c)(d-b)(d-a)(c-b)(c-a)(b-a).\]

Un autre exemple, déjà vu :

Soit \(a,b,c,x\) quatre complexes. Factoriser \[P_{a,b,c}(x) = \begin{vmatrix} x & a & b & c \\ a & x & c & b \\ b & c & x & a \newline c & b & a & x \end{vmatrix}.\]
Après une bonne contemplation, il saute aux yeux que : \[\begin{pmatrix} x & a & b & c \\ a & x & c & b \\ b & c & x & a \\ c & b & a & x \end{pmatrix} \begin{pmatrix} 1 & -1 & 1 & -1 \\ 1 & -1 & -1 & 1 \\ 1 & 1 & 1 & 1 \\ 1 & 1 & -1 & -1 \end{pmatrix} = \begin{pmatrix} x+c+b+a & -x+c+b-a & x-c+b-a & -x-c+b+a \\ x+c+b+a & -x+c+b-a & -x+c-b+a & x+c-b-a \\ x+c+b+a & x-c-b+a & x-c+b-a & x+c-b-a \\ x+c+b+a & x-c-b+a & -x+c-b+a & -x-c+b+a \end{pmatrix}\] d’où l’égalité : \[P_{a,b,c}(x) \begin{vmatrix} 1 & -1 & 1 & -1 \\ 1 & -1 & -1 & 1 \\ 1 & 1 & 1 & 1 \\ 1 & 1 & -1 & -1 \end{vmatrix} = (x+a+b+c)(x+a-b-c)(x-a+b-c)(x-a-b+c) \begin{vmatrix} 1 & -1 & 1 & -1 \\ 1 & -1 & -1 & 1 \\ 1 & 1 & 1 & 1 \\ 1 & 1 & -1 & -1 \end{vmatrix}\] et il suffit de vérifier que \(\begin{vmatrix} 1 & -1 & 1 & -1 \\ 1 & -1 & -1 & 1 \\ 1 & 1 & 1 & 1 \newline 1 & 1 & -1 & -1 \end{vmatrix}\) est non nul. Or ses vecteurs colonnes sont orthogonaux deux à deux.

Enfin un exemple qui ressemble plus à une devinette :

Démontrer que \[\begin{array}{ccc}\left|\begin{array}{ccc}1-ps & pr-q & qs-r\\\\ ns-m & 1-nr & mr-s\\\\ mp-n & nq-p & 1-mq\end{array}\right|=\left|\begin{array}{ccc}1 & m & n\\\\ q & 1 & p\\\\ r & s & 1\end{array}\right|^{2}=\left|\begin{array}{cccc}1+m^{2}+n^{2}& m+q+np & n+r+ms\\\\ m+q+np & 1+p^{2}+q^{2}& p+s+qr\\newline\ n+r+ms & p+s+qr & 1+r^{2}+s^{2}\end{array}\right|\end{array}\]
Soit \(A = \begin{pmatrix} 1 & m & n\\ q & 1 & p\\ r & s & 1 \end{pmatrix}\). La deuxième égalité vient de \[A^tA = \left(\begin{array}{ccc}1 & m & n\\\\ q & 1 & p\\\\ r & s & 1\end{array}\right)\times \left(\begin{array}{ccc}1 & q & r\\\\ m & 1 & s\\\\ n & p & 1\end{array}\right) = \left(\begin{array}{cccc}1+m^{2}+n^{2}& m+q+np & n+r+ms\\\\ m+q+np & 1+p^{2}+q^{2}& p+s+qr\\\\ n+r+ms & p+s+qr & 1+r^{2}+s^{2}\end{array}\right).\] Pour la première, on commence par calculer les cofacteurs : \[\mathop{\rm det}A I_3 = A \left(\begin{array}{ccc}1-ps & ns-m & mp-n \\\\ pr-q & 1-nr & 1-mq\\\\ qs-r & nq-p & mr-s \end{array}\right).\] En calculant le déterminant des deux membres : \[(\mathop{\rm det}A)^3 = \mathop{\rm det}A \left|\begin{array}{ccc}1-ps & ns-m & mp-n \\\\ pr-q & 1-nr & 1-mq\\\\ qs-r & nq-p & mr-s \end{array}\right| = \mathop{\rm det}A \left|\begin{array}{ccc}1-ps & pr-q & qs-r\\\\ ns-m & 1-nr & mr-s\\\\ mp-n & nq-p & 1-mq\end{array}\right|\] puisqu’une matrice a même déterminant que sa transposée. Lorsque \(A\) est inversible, on obtient la première égalité en simplifiant par \(\mathop{\rm det}A\). Lorsque \(A\) n’est pas inversible, la matrice transposée des cofacteurs n’est pas inversible non plus. En effet, lorsqu’on la multiplie par \(A\) on trouve la matrice nulle. Donc les deux membres de l’égalité sont nuls.

Polynôme caractéristique

Cette méthode sera utilisée l’an prochain.

Bibliographie


    Barre utilisateur

    [ID: 82] [Date de publication: 6 janvier 2022 17:01] [Catégorie(s): Le cours de SUP ] [ Nombre commentaires: 0] [nombre d'éditeurs: 1 ] [Editeur(s): Emmanuel Vieillard-Baron ] [nombre d'auteurs: 3 ] [Auteur(s): Emmanuel Vieillard-Baron Alain Soyeur François Capaces ]




    Commentaires sur le cours

    Documents à télécharger

    L'article complet